Diabetes, chapter 48 diabetes, Chapter 48 Diabetes Mellitus, Diabetes Mellitus NCLEX

Ace your homework & exams now with Quizwiz!

Hemoglobin A1C (Hb A1C) range

5.7-6.4%

Approximate Normal range for glucose

70-120mg/dL

Criteria for high risk pregnancy are (select all that apply): A. Obesity B. Prior history of gestational diabetes C. Presence of glucouria D. Mood swings E. Strong family history of type 2 diabetes F. Diagnosis of polycystic ovary syndrome

A, B C, E, F

4. The nurse is assigned to the care of a 64-year-old patient diagnosed with type 2 diabetes. In formulating a teaching plan that encourages the patient to actively participate in management of the diabetes, what should be the nurse's initial intervention? A. Assess patient's perception of what it means to have diabetes. B. Ask the patient to write down current knowledge about diabetes. C. Set goals for the patient to actively participate in managing his diabetes. D. Assume responsibility for all of the patient's care to decrease stress level

A. Assess patient's perception of what it means to have diabetes. Rationale. In order for teaching to be effective, the first step is to assess the patient. Teaching can be individualized once the nurse is aware of what a diagnosis of diabetes means to the patient. After the initial assessment, current knowledge can be assessed, and goals should be set with the patient. Assuming responsibility for all of the patient's care will not facilitate the patient's health.

9. The nurse is assisting a patient with newly diagnosed type 2 diabetes to learn dietary planning as part of the initial management of diabetes. The nurse would encourage the patient to limit intake of which foods to help reduce the percent of fat in the diet? A. Cheese B. Broccoli C. Chicken D. Oranges

A. Cheese Rationale. Cheese is a product derived from animal sources and is higher in fat and calories than vegetables, fruit, and poultry. Excess fat in the diet is limited to help avoid macrovascular changes.

6. The nurse is evaluating a 45-year-old patient diagnosed with type 2 diabetes mellitus. Which symptom reported by the patient is considered one of the classic clinical manifestations of diabetes? A. Excessive thirst B. Gradual weight gain C. Overwhelming fatigue D. Recurrent blurred vision

A. Excessive thirst Rationale. The classic symptoms of diabetes are polydipsia (excessive thirst), polyuria, (excessive urine output), and polyphagia (increased hunger). Weight gain, fatigue, and blurred vision may all occur with type 2 diabetes, but are not classic manifestations.

10. Laboratory results have been obtained for a 50-year-old patient with a 15-year history of type 2 diabetes. Which result reflects the expected pattern accompanying macrovascular disease as a complication of diabetes? A. Increased triglyceride levels B. Increased high-density lipoproteins (HDL) C. Decreased low-density lipoproteins (LDL) D. Decreased very-low-density lipoproteins (VLDL)

A. Increased triglyceride levels Rationale. Macrovascular complications of diabetes include changes to large- and medium-sized blood vessels. They include cerebrovascular, cardiovascular, and peripheral vascular disease. Increased triglyceride levels are associated with these macrovascular changes. Increased HDL, decreased LDL, and decreased VLDL are positive in relation to atherosclerosis development.

Which patient with type 1 diabetes mellitus would be at the highest risk for developing hypoglycemic unawareness? A. A 58-year-old patient with diabetic retinopathy B. A 73-year-old patient who takes propranolol (Inderal) C. A 19-year-old patient who is on the school track team D. A 24-year-old patient with a hemoglobin A1C of 8.9%

B. A 73-year-old patient who takes propranolol (Inderal) Rationale. Hypoglycemic unawareness is a condition in which a person does not experience the warning signs and symptoms of hypoglycemia until the person becomes incoherent and combative or loses consciousness. Hypoglycemic awareness is related to autonomic neuropathy of diabetes that interferes with the secretion of counterregulatory hormones that produce these symptoms. Older patients and patients who use â-adrenergic blockers (e.g., propranolol) are at risk for hypoglycemic unawareness.

7. A 51-year-old patient with diabetes mellitus is scheduled for a fasting blood glucose level at 8:00 AM. The nurse instructs the patient to only drink water after what time? A. 6:00 PM on the evening before the test B. Midnight before the test C. 4:00 AM on the day of the test D. 7:00 AM on the day of the test

B. Midnight before the test Rationale. Typically, a patient is ordered to be NPO for 8 hours before a fasting blood glucose level. For this reason, the patient who has a lab draw at 8:00 AM should not have any food or beverages containing any calories after midnight.

13. The patient received regular insulin 10 units subcutaneously at 8:30 PM for a blood glucose level of 253 mg/dL. The nurse plans to monitor this patient for signs of hypoglycemia at which time related to the insulin's peak action? A. 8:40 PM to 9:00 PM B. 9:00 PM to 11:30 PM C. 10:30 PM to 1:30 AM D. 12:30 AM to 8:30 AM

C. 10:30 PM to 1:30 AM Rationale. Regular insulin exerts peak action in 2 to 5 hours, making the patient most at risk for hypoglycemia between 10:30 PM and 1:30 AM. Rapid-acting insulin's onset is between 10-30 minutes with peak action and hypoglycemia most likely to occur between 9:00 PM and 11:30 PM. With intermediate acting insulin, hypoglycemia may occur from 12:30 AM to 8:30 AM.

What is the priority action for the nurse to take if the patient with type 2 diabetes complains of blurred vision and irritability? A. Call the physician B. Administer insulin as ordered C. Check the patient's blood glucose level D. Assess for other neurologic symptoms

C. Check the patient's blood glucose level Rationale. Blood glucose testing should be performed whenever hypoglycemia is suspected so that immediate action can be taken if necessary.

14. A college student is newly diagnosed with type 1 diabetes. She now has a headache, changes in her vision, and is anxious, but does not have her portable blood glucose monitor with her. Which action should the campus nurse advise her to take? A. Eat a piece of pizza. B. Drink some diet pop. C. Eat 15 g of simple carbohydrates. D. Take an extra dose of rapid-acting insulin.

C. Eat 15 g of simple carbohydrates. Rationale. When the patient with type 1 diabetes is unsure about the meaning of the symptoms she is experiencing, she should treat herself for hypoglycemia to prevent seizures and coma from occurring. She should also be advised to check her blood glucose as soon as possible. The fat in the pizza and the diet pop would not allow the blood glucose to increase to eliminate the symptoms. The extra dose of rapid-acting insulin would further decrease her blood glucose.

What is a finding in DKA that is not seen in hyperosmolar hyperglycemic syndrome (HHS)? A. Glucose level above 400 mg/dL B. Hyperkalemia C. Ketones in blood D. Urine output of 30 mL/hr

C. Ketones in blood Rationale. The main difference between the two conditions is that ketone bodies are absent or minimal in HHS because the body has enough insulin to prevent ketoacidosis. Both have high glucose levels, although the level in HHS tends to be higher (above 600 mg/dL). Hypokalemia is possible in both, although it is more likely and serious in DKA. Urine output of 30 mL/hr is normal obligatory output; both conditions are likely to have dehydration and decreased output.

You are teaching a 54-year-old patient with diabetes about proper composition of the daily diet. You explain that the guideline for carbohydrate intake is A. 80% of daily intake. B. minimum of 80 g/day. C. minimum of 130 g/day. D. maximum of 130 g/day.

C. minimum of 130 g/day Rationale. The recommendation for carbohydrate intake is a minimum of 130 g/day. Low-carbohydrate diets are not recommended for diabetes management.

The patient in the emergency department is diagnosed with diabetic ketoacidosis. Which laboratory value is essential for you to monitor? A. Magnesium (Mg) B. Hemoglobin (Hb) C. White blood cells (WBCs) D. Potassium (K)

D. Potassium (K) Rationale. Even if the patient has normal potassium levels, there can be significant hypokalemia when insulin is administered as it pushes the serum potassium intracellularly. This can lead to life-threatening hypokalemia. The other options are not as significant.

Rapid onset Type 1 or type 2?

Type 1

Requires exogenous insulin to sustain life Type 1 or Type 2?

Type 1

Switches energy source to fat/protein which results in fatigue, weakness and weight loss Type 1 or Type 2?

Type 1

Gradual onset

Type 2

Nonspecific symptoms Type 1 or Type 2?

Type 2

Pancreas continues to produce some endogenous insulin but insulin produced is insufficient or is poorly utilized by tissues Type 1 or Type 2?

Type 2

Prolonged wound healing and recurrent infections Type 1 or 2?

Type 2

Recurrent infections

Type 2

Visual changes Tupe 1 or Type 2?

Type 2

List the 4 injection sites

1. Abdomen 2. Arm 3. Thigh 4.Buttocks

Long term complications of diabetes

1. End stage renal disease 2. Lower limb Amputation 3. Eye Complications

What are 3 other types of diabetes?

1. Gestational 2. Prediabetes 3. Secondary diabetes

4 examples of counterregulatory hormones

1. Glucagon 2. Epinephrine 3. Cortisol 4. Growth hormone

proliferative and nonproliferative

2 types of diabetic retinopathy are

polyuria, polydipsia, and polyphagia

3 classic P's of diabetes

The elderly patient with type 2 diabetes mellitus presents to the clinic with a fever and productive cough. The diagnosis of pneumonia is made. You notice tenting skin, deep tongue furrows, and vital signs of 110/80 mm Hg, 120 beats/minute, and 24 breaths/minute. What assessment is important for you to obtain? A. Blood glucose B. Orthostatic blood pressures C. Urine ketones D. Temperature

A. Blood glucose Rationale. HHS is typically seen in patients with type 2 diabetes and infection, such as pneumonia. The main presenting sign is a glucose level above 600 mg/dL. Enough evidence of dehydration already exists that orthostatic vital sign assessments are not a priority, and they are often inaccurate in the elderly due to poor vascular tone. Patients with HHS do not have elevated ketone levels, which is a key distinction between HHS and DKA. Temperature will eventually be taken but is often blunted in the elderly and diabetics. An infectious diagnosis has already been made. The glucose level for appropriate fluid and insulin treatment is the priority.

3.The nurse has been teaching a patient with diabetes mellitus how to perform self-monitoring of blood glucose (SMBG). During evaluation of the patient's technique, the nurse identifies a need for additional teaching when the patient does what? A. Chooses a puncture site in the center of the finger pad. B. Washes hands with soap and water to cleanse the site to be used. C. Warms the finger before puncturing the finger to obtain a drop of blood. D. Tells the nurse that the result of 110 mg/dL indicates good control of diabetes.

A. Chooses a puncture site in the center of the finger pad. Rationale. The patient should select a site on the sides of the fingertips, not on the center of the finger pad as this area contains many nerve endings and would be unnecessarily painful. Washing hands, warming the finger, and knowing the results that indicate good control all show understanding of the teaching.

The nurse receives report on a 52-year-old client with type 2 diabetes: Physical Assessment Diagnostic Findings Provider Prescriptions Lungs clear Glucose 179 mg/dL Regular insulin 8 units if blood glucose 250 to 275 mg/dL and cold to touch Right great toe mottled Hemoglobin A1c 6.9% Regular insulin 10 units if glucose 275 to 300 mg/dL Client states wears eyeglasses to read Which complication of diabetes does the nurse report to the provider? Poor glucose control Visual changes Respiratory distress Decreased peripheral perfusion Correct

A cold, mottled toe may indicate arterial occlusion secondary to arterial occlusive disease or embolization; this must be reported to avoid potential gangrene and amputation. Although one glucose reading is elevated, the hemoglobin A1c indicates successful glucose control over the past 3 months. After the age of 40, reading glasses may be needed due to difficulty in accommodating to close objects. Lungs are clear and no evidence of distress is noted.

A client has just been diagnosed with diabetes. Which factor is most important for the nurse to assess in the client before providing instruction about the disease and its management? Current lifestyle Educational and literacy level Correct Sexual orientation Current energy level

A large amount of information must be synthesized; typically written instructions are given. The client's educational and literacy level is essential information. Although lifestyle should be taken into account, it is not the priority. Sexual orientation will have no bearing on the ability of the client to provide self-care. Although energy level will influence the ability to exercise, it is not essential.

Which is a description of acanthosis nigricans, an integumentary complication of diabetes mellitus? A. Dark, coarse, thickened skin predominantly on the neck B. Widespread redness in the lower extremities C. Unilateral, clustered skin vesicles along a peripheral sensory nerve D. Threadlike, brownish, linear burrows between the fingers and inner wrists

A. Dark, coarse, thickened skin predominantly on the neck Rationale. Acanthosis nigricans is characterized by dark, coarse, thickened skin predominantly in the flexures and on the neck. Widespread redness is a sign of cellulitis. Unilateral, clustered skin vesicles along a peripheral sensory nerve describe herpes zoster (shingles). Threadlike, brownish, linear burrows between fingers and inner wrists describe scabies.

The patient with type 1 diabetes mellitus is diaphoretic and shaky, and he reports feeling very lightheaded. There is no readily accessible blood glucose monitoring equipment. What action should you take? A. Give 8 oz of milk. B. Give 6 oz of juice with four packets of sugar in the juice. C. Wait to act until there is confirmation of the blood glucose level. D. Administer the patient's insulin.

A. Give 8 oz of milk. Rationale. Hypoglycemia is treated with 15 to 20 grams of simple (fast-acting) carbohydrate, such as 8 ounces of low-fat milk. Avoid overtreatment with large quantities of quick-acting carbohydrates that can cause a rapid fluctuation to hyperglycemia. If the patient has manifestations of hypoglycemia and monitoring equipment is not available, hypoglycemia should be assumed and treatment should be initiated. The patient has signs of hypoglycemia and insulin is not indicated.

You are caring for a patient with newly diagnosed type I diabetes. What information is essential to include in your patient teaching before discharge from the hospital? (Select all that apply) A. Insulin administration B. Elimination of sugar from diet C. Need to reduce physical activity D. Use of portable blood glucose monitor E. Hyperglycemia prevention, symptoms, and treatment

A. Insulin administration D. Use of portable blood glucose monitor E. Hyperglycemia prevention, symptoms, and treatment Rationale. The nurse ensures that the patient understands the proper use of insulin. The nurse teaches the patient how to use the portable blood glucose monitor and how to recognize and treat signs and symptoms of hypoglycemia and hyperglycemia.

In the first hour or two after meals, insulin concentrations_____rapidly in blood A. Rises B. Decreases

A. Rises

The patient has a 3:00 AM blood glucose level of 50 mg/dL and a 7:00 AM glucose level of 150 mg/dL. What is the proper explanation of these findings and anticipated intervention? A. Somogyi effect with need for less insulin at night B. Somogyi effect with need for a snack at 3:00 AM C. Dawn phenomenon with need for more insulin in the morning D. Dawn phenomenon with need for less food in the morning

A. Somogyi effect with need for less insulin at night Rationale. Somogyi effect is rebound caused by too much insulin at bedtime. The hypoglycemia produces counterregulatory hormones, causing rebound hyperglycemia. It is treated with less insulin at night. The dawn phenomenon is hyperglycemia in the morning due to release of counterregulatory hormones (e.g., growth hormone, cortisol) in the predawn hours. It is treated by increasing the insulin dose and an appropriate bedtime snack.

Which of these clients with diabetes does the endocrine unit charge nurse assign to an RN who has floated from the labor/delivery unit? A 58-year-old with sensory neuropathy who needs teaching about foot care A 68-year-old with diabetic ketoacidosis who has an IV running at 250 mL/hr A 70-year-old who needs blood glucose monitoring and insulin before each meal Correct A 76-year-old who was admitted with fatigue and shortness of breath

A nurse from the labor/delivery unit would be familiar with blood glucose monitoring and insulin administration because clients with type 1 and gestational diabetes are frequently cared for in the labor/delivery unit. The 58-year-old with sensory neuropathy, the 68-year-old with diabetic ketoacidosis, and the 76-year-old with fatigue and shortness of breath all have specific teaching or assessment needs that are better handled by nurses more familiar with caring for older adults with diabetes.

Which statement by the patient with type 2 diabetes is accurate? A. " I am supposed to have a meal or snack if I drink alcohol" B. "I am not allowed to eat any sweets because of my diabetes" C. "I do not need to watch what I eat because my diabetes is not the bad kind". D. "The amount of fat in my diet is not important. Only carbohydrates raise my blood sugar."

A. " I am supposed to have a meal or snack if I drink alcohol". Rationale. Alcohol should be consumed with food to reduce the risk of hypoglycemia.

The nurse teaches a 38-year-old man who was recently diagnosed with type 1 diabetes mellitus about insulin administration. Which statement by the patient requires an intervention by the nurse? A. "I will discard any insulin bottle that is cloudy in appearance." B. "The best injection site for insulin administration is in my abdomen." C. "I can wash the site with soap and water before insulin administration." D. "I may keep my insulin at room temperature (75o F) for up to a month."

A. "I will discard any insulin bottle that is cloudy in appearance." Rationale. Intermediate-acting insulin and combination premixed insulin will be cloudy in appearance. Routine hygiene such as washing with soap and rinsing with water is adequate for skin preparation for the patient during self-injections. Insulin vials that the patient is currently using may be left at room temperature for up to 4 weeks unless the room temperature is higher than 86° F (30° C) or below freezing (less than 32° F [0° C]). Rotating sites to different anatomic sites is no longer recommended. Patients should rotate the injection within one particular site, such as the abdomen.

The nurse is reviewing laboratory results for the clinic patients to be seen today. Which patient meets the diagnostic criteria for diabetes mellitus? A. A 48-year-old woman with a hemoglobin A1C of 8.4% B. A 58-year-old man with a fasting blood glucose of 111 mg/dL C. A 68-year-old woman with a random plasma glucose of 190 mg/dL D. A 78-year-old man with a 2-hour glucose tolerance plasma glucose of 184 mg/dL

A. A 48-year-old woman with a hemoglobin A1C of 8.4% Rationale. Criteria for a diagnosis of diabetes mellitus include a hemoglobin A1C ≥ 6.5%, fasting plasma glucose level =126 mg/dL, 2-hour plasma glucose level =200 mg/dL during an oral glucose tolerance test, or classic symptoms of hyperglycemia or hyperglycemic crisis with a random plasma glucose =200 mg/dL.

Which is the best referral that the community health nurse can suggest to a client who has been newly diagnosed with diabetes? a. American Diabetes Association (ADA) b. Centers for Disease Control and Prevention c. Primary health care provider office d. Pharmaceutical representative

A. American Diabetes Association (ADA)

Which symptom would make a clinic nurse suspect undiagnosed diabetes mellitus in a patient? A. Balanitis B. Blepharoptosis C. "Mole" with waxy border and recent central crater D. Nits on the hair shaft with pruritus

A. Balanitis Rationale. Inflammation of the skin covering the glans penis is seen in uncircumcised males with undiagnosed diabetes mellitus as a result of glucosuria. Blepharoptosis is drooping of the upper eyelid and is not directly related to diabetes mellitus. A "mole" with waxy border and recent central crater is a description of basal cell carcinoma, which is not directly related to diabetes mellitus. Nits on the hair shaft with pruritus is a description of pediculosis capitis (lice).

when the pt is rehydrated, the excess K+ will come out in the urine

PT's with DKA should not be given medication to lower the potassium level because

spontaneously

Pain of sensory polyneuropathy is often worst at night and usually resolves

heart and kidney disease, blindness, amputation, and complications in pregnancy

People with diabetes are at an increased risk for

The nurse instructs a 22-year-old female patient with diabetes mellitus about a healthy eating plan. Which statement made by the patient indicates that teaching was successful? A. "I plan to lose 25 pounds this year by following a high-protein diet." B. "I may have a hypoglycemic reaction if I drink alcohol on an empty stomach." C. "I should include more fiber in my diet than a person who does not have diabetes." D. "If I use an insulin pump, I will not need to limit the amount of saturated fat in my diet."

B. "I may have a hypoglycemic reaction if I drink alcohol on an empty stomach." Rationale. The risk for alcohol-induced hypoglycemia is reduced by eating carbohydrates when drinking alcohol. Intensified insulin therapy, such as the use of an insulin pump, allows considerable flexibility in food selection and can be adjusted for alterations from usual eating and exercise habits. However, saturated fat intake should still be limited to less than 7% of total daily calories. Daily fiber intake of 14 g/1000 kcal is recommended for the general population and for patients with diabetes mellitus. High-protein diets are not recommended for weight loss.

Which explanation best assists a client in differentiating type 1 diabetes from type 2 diabetes? Most clients with type 1 diabetes are born with it. People with type 1 diabetes are often obese. Those with type 2 diabetes make insulin, but in inadequate amounts. Correct People with type 2 diabetes do not develop typical diabetic complications.

People with type 2 diabetes make some insulin but in inadequate amounts, or they have resistance to existing insulin. Although type 1 diabetes may occur early in life, it may be caused by immune responses. Obesity is typically associated with type 2 diabetes. People with type 2 diabetes are at risk for complications, especially cardiovascular complications.

You taught a patient admitted with diabetes, cellulitis, and osteomyelitis the principles of foot care. You confirm that the patient understands the principles of foot care if he makes which statement? A. "I should walk barefoot only in nice dry weather." B. "I should look at the condition of my feet every day." C. "I am lucky my shoes fit so nice and tight because they give me firm support." D. "When I am allowed up out of bed, I should check the shower water with my toes."

B. "I should look at the condition of my feet every day." Rationale. Patients with diabetes mellitus need to inspect the feet daily for broken areas that are at risk for delayed wound healing. Water temperature should be tested with the hands first. Properly fitted (not tight) shoes should be worn at all times.

11. The nurse has taught a patient admitted with diabetes, cellulitis, and osteomyelitis about the principles of foot care. The nurse evaluates that the patient understands the principles of foot care if the patient makes what statement? A. "I should only walk barefoot in nice dry weather." B. "I should look at the condition of my feet every day." C. "I am lucky my shoes fit so nice and tight because they give me firm support." D. "When I am allowed up out of bed, I should check the shower water with my toes."

B. "I should look at the condition of my feet every day." Rationale. Patients with diabetes mellitus need to inspect their feet daily for broken areas that are at risk for infection and delayed wound healing. Properly fitted (not tight) shoes should be worn at all times. Water temperature should be tested with the hands first.

The nurse caring for four diabetic clients has all of these activities to perform. Which is appropriate to delegate to unlicensed assistive personnel (UAP)? Perform hourly bedside blood glucose checks for a client with hyperglycemia. Correct Verify the infusion rate on a continuous infusion insulin pump. Monitor a client with blood glucose of 68 mg/dL for tremors and irritability. Check on a client who is reporting palpitations and anxiety.

Performing bedside glucose monitoring is an activity that may be delegated because it does not require extensive clinical judgment to perform; the nurse will follow up with the results. Intravenous therapy and medication administration are not within the scope of practice for UAP. The client with blood glucose of 68 mg/dL will need further monitoring, assessment, and intervention not within the scope of practice for UAP. The client reporting palpitations and anxiety may have hypoglycemia, requiring further intervention; this client must be assessed by licensed nursing staff.

autonomic nerves

Polyneuropathy involves sensory and

occlusionof the tubin or the needle and infection at the site of injection

Potential problems with insulin pump are?

growth of abnormal capillaries on the retina and optic disc. The fragile vessels can penetrate the vitreous humor and rupture

Proliferative disease is characterized by

acidosis with a pH of less than 7.3 or a bicarbonate level less than 15

Pt's with ketoacidosis have a glucose level above 200 and

Type 1 has 12 x's more of a chance

Pt's with type 1 or 2 diabetes have more of a chance of developing esrd?

Types of Meglitinides

Rapaglinide (Prandin) Nateglinide (Starlix)

Inhaled Insulin

Rapid acting human insulin that can be absorbed in the lungs. Afrezza available in the U.S. in 2015, must be used with injectable. (types: Afrezza) fine powder delivered through nebulizer, rapid absorption

insulin lispro (Humalog), insulin aspart (novalog), and insulin glulisine (adpidra)

Rapid acting insulin ex

Purpose for Biguanide (4)

Reduce glucose production by liver Increase glucose sensitivity at tissues Increase glucose transport to cells Lowers lipid levels

Genetic Link: Type 1 Diabetes

Related to human leukocyte antigens (HLAs).

neuroglucopenic

S&S's of hypoglycemia can be adrenergic or

persistent proteinuria, high bp ad serum creatinine, hematuria, and oliguria or anuria

S&S's of kidney failure

Self monitoring blood glucose readings (SMBG) vs Lab work readings

SMBG takes blood from the capillaries (whole blood) Lab work from hospitals will use venous blood Venous blood will show higher results than capillary blood. Normally 10-12% higher than cap blood. This is normal.

both legs symmetrically

Sensory polyneuropathy commonly affects

The nurse is providing discharge teaching to a client with diabetes about injury prevention for peripheral neuropathy. Which statement by the client indicates a need for further teaching? "I can break in my shoes by wearing them all day." Correct "I need to monitor my feet daily for blisters or skin breaks." "I should never go barefoot." "I should quit smoking."

Shoes should be properly fitted and worn for a few hours a day to break them in, with frequent inspection for irritation or blistering. People with diabetes have decreased peripheral circulation, so even small injuries to the feet must be managed early. Going barefoot is contraindicated. Tobacco use further decreases peripheral circulation in a client with diabetes.

regular insulin (Humulin R, and Novolin R)

Short acting insulin ex

when the pt has rebound hyperglycemia in response to hypoglycemia.

Sogoymi phenomenom

Noninsulin Injectible Agents

Stimulates the release of insulin, decreases glucagon secretion, and slow gastric emptying, increase satiety.

A client expresses fear and anxiety over the life changes associated with diabetes, stating, "I am scared I can't do it all and I will get sick and be a burden on my family." What is the nurse's best response? "It is overwhelming, isn't it?" "Let's see how much you can learn today, so you are less nervous." "Let's tackle it piece by piece. What is most scary to you?" Correct "Other people do it just fine."

Suggesting the client tackle it piece by piece and asking what is most scary to him or her is the best response; this approach will allow the client to have a sense of mastery with acceptance. Referring to the illness as overwhelming is supportive, but is not therapeutic or helpful to the client. Trying to see how much the client can learn in one day may actually cause the client to become more nervous; an overload of information is overwhelming. Suggesting that other people handle the illness just fine is belittling and dismisses the client's concerns.

stimulating the pancreas to secrete more insulin

Sulfonylureas decrease the blood sugars by

range from tingling, numbness, and burning to complete loss of sensation.

Symptoms of sensory polyneuropathy

blood stream

THe excess fluid isn't passed into the body tissues, it's retained in the

adequate urine is established.

THe only time replacement K+ is given is once

A client recently admitted with new-onset type 2 diabetes will be discharged with a self-monitoring blood glucose machine. When is the best time for the nurse to explain to the client the proper use of the machine? Day of discharge On admission When the client states readiness While performing the test in the hospital Correct

Teaching the client about the operation of the machine while performing the test in the hospital is the best way for the client to learn. The teaching can be reinforced before discharge. Instructing the client on the day of admission or the day of discharge would be overwhelming to the client because of all of the other activities taking place on those days. The client may never feel ready to learn this daunting task; the nurse must be more proactive.

Glycemic Index (GI)

Term used to describe rise in blood glucose levels 2 hours after carbohydrate-containing food is consumed

Which is the best referral that the nurse can suggest to a client who has been newly diagnosed with diabetes? American Diabetes Association Correct Centers for Disease Control and Prevention Health care provider office Pharmaceutical representative

The American Diabetes Association can provide national and regional support and resources to clients with diabetes and their families. The Centers for Disease Control and Prevention does not focus on diabetes. The client's health care provider's office is not the best resource for diabetes information and support. A pharmaceutical representative is not an appropriate resource for diabetes information and support.

ketonuria, kussmaul resp, orthostatic hypotension, htn, nausea, vomiting, and lethargy or change in consciousness

The assessment of a pt with ketoacidosis is likely to show

A client newly diagnosed with diabetes is not ready or willing to learn diabetes control during the hospital stay. Which information is the priority for the nurse to teach the client and the client's family? Causes and treatment of hyperglycemia Causes and treatment of hypoglycemia Correct Dietary control Insulin administration

The causes and treatment of hypoglycemia must be understood by the client and family to manage the client's diabetes effectively. The causes and treatment of hyperglycemia is a topic for secondary teaching and is not the priority for the client with diabetes. Dietary control and insulin administration are important, but are not the priority in this situation.

The nurse has just taken change-of-shift report on a group of clients on the medical-surgical unit. Which client does the nurse assess first? Client taking repaglinide (Prandin) who has nausea and back pain Client taking glyburide (Diabeta) who is dizzy and sweaty Correct Client taking metformin (Glucophage) who has abdominal cramps Client taking pioglitazone (Actos) who has bilateral ankle swelling

The client taking glyburide (Diabeta) who is dizzy and sweaty has symptoms consistent with hypoglycemia and should be assessed first because this client displays the most serious adverse effect of antidiabetic medications. Although the client taking repaglinide who has nausea and back pain requires assessment, the client taking glyburide takes priority. Metformin may cause abdominal cramping and diarrhea, but the client taking it does not require immediate assessment. Ankle swelling is an expected side effect of pioglitazone.

A client with type 1 diabetes arrives in the emergency department breathing deeply and stating, "I can't catch my breath." The client's vital signs are: T 98.4° F (36.9° C), P 112 beats/min, R 38 breaths/min, BP 91/54 mm Hg, and O2 saturation 99% on room air. Which action does the nurse take first? Check the blood glucose. Correct Administer oxygen. Offer reassurance. Attach a cardiac monitor.

The client's clinical presentation is consistent with diabetic ketoacidosis, so the nurse should initially check the client's glucose level. Based on the oxygen saturation, oxygen administration is not necessary. The nurse provides support, but it is early in the course of assessment and intervention to offer reassurance without more information. Cardiac monitoring may be implemented, but the first action should be to obtain the glucose level.

a pt meets one or more of the following criteria on more than 1 occasion- fasting sugars above 126, A1c above 6.5, syptoms of dm in addition to a random sugar check above 200, 2 hour postprandial glucose above 200 in a glucose tolerance test

The diagnosis of DM is made when

potassium

The electrolyte is dka that is of primary imp is

1/3. which triples the production of lipoproteins inc the chance of atherosclerosis

The liver can store up to ____ of it's weight as fat

A client with type 2 diabetes who is taking metformin (Glucophage) is seen in the diabetic clinic. The fasting blood glucose is 108 mg/dL, and the glycosylated hemoglobin (HbA1C) is 8.2%. Which action does the nurse plan to take next? Instruct the client to continue with the current diet and metformin use. Discuss the need to check blood glucose several times every day. Talk about the possibility of adding rapid-acting insulin to the regimen. Ask the client about current dietary intake and medication use. Correct

The nurse's first action should be to assess whether the client is adherent to the currently prescribed diet and medications. The client's current diet and medication use have not been successful in keeping glucose in the desired range. Checking blood glucose more frequently and/or using rapid-acting insulin may be appropriate, but this will depend on the assessment data. The HbA1C indicates that the client's average glucose level is not in the desired range, but discussing the need to check blood glucose several times every day assumes that the client is not compliant with the therapy and glucose monitoring. The nurse should not assume that adding insulin, which must be prescribed by the provider, is the answer without assessing the underlying reason for the treatment failure.

Which action is correct when drawing up a single dose of insulin? Wash hands thoroughly and don sterile gloves. Shake the bottle of insulin vigorously to mix the insulin. Pull back plunger to draw air into the syringe equal to the insulin dose. Correct Recap the needle and save the syringe for the next dose of insulin.

The plunger is pulled back to draw an amount of air into the syringe that is equal to the insulin dose. The air is then injected into the insulin bottle before withdrawing the insulin dose. Although handwashing is important before any medication administration, sterile gloves are not required. The bottle of insulin should be rolled gently in the palms of the hands to mix the insulin, not shaken. Insulin syringes are never recapped or reused; the syringe and needle should be disposed of (without recapping) in a puncture-proof container.

kineys and eyes

The things most often affected by microvascular affects are

brain, nerves, heart, and lens of the eye

Tissues that can use glucose without insulin are

The nurse is teaching a client with diabetes about proper foot care. Which statement by the client indicates that teaching was effective? "I should go barefoot in my house so that my feet are exposed to air." "I must inspect my shoes for foreign objects before putting them on." Correct "I will soak my feet in warm water to soften calluses before trying to remove them." "I must wear canvas shoes as much as possible to decrease pressure on my feet."

To avoid injury or trauma to the feet, shoes should be inspected for foreign objects before they are put on. Diabetic clients should not go barefoot because foot injuries can occur. To avoid injury or trauma, a callus should be removed by a podiatrist, not by the client. The diabetic client must wear firm support shoes to prevent injury.

28 calories/kg of body weight.

To maintain weight you consume

15-10 calories/kg of body weight

To reduce weight you consume

15-20 g of quick acting carbs and repeat q 15-30 until their blood sugar is above 70

To treat hypoglycemia in a conscious pt you administer

genetic, immunologic, and environmental factors such as viruses and toxins

Type 1 DM has been attributed to

Recurrent vaginal yeast or candidate infections

Type 2

inadequate endogenous insulin and the body's inability to use insulin properly

Type 2 DM is characterized by

obesity, sedentary lifestyle, fam hx of dm, over 40, hx of delivering a baby over 10 lbs, ethnicity, PCOS, htn and heart disease, presence of acanthosis nigricans or other cond assoc with insulin resistance

Type 2 risk factors

atonic bladder which is where bladder capacity increases causing retention with overflow, ED and retrograde ejaculation in men, and painful intercourse in women

Urinary problems related to autonomic neuropathy

Insulin Pump

Used to deliver insulin to patients with diabetes Programmed by the user to deliver measured doses of insulin throughout the day

Clear to cloudy

WHat order do you drop up insulins?

every 2-3 days

WIth an insulin pump how often do you change the supplies?

heat, massage, and exercise

What increases the rate of absorption of insulin?

Lantus and Levemir (Long acting)

What insulins can't be mixed with anything?

beta blockers

What medication can mask the adrenergic symptoms of hypoglycemia?

ASA, and ace inhibitors

What medications are advised for pt's with risk of heart and renal disease?

blood stream.

When insulin is absent, glucose can't enter most cells so it remains in the

when the pt has nausea, vomiting, and abdominal pain.

When is ketone testing recommended?

When the pt reports awakening with HA's, and c/o restless sleep, nightmares, enuresis, and nausea and vomitting

When should the sogoymi phenomenom be suspected?

thirst center causing the pt to have polydipsia

When the blood becomes thick with glucose, which increases the osmality of the blood, which stimulates the

1) presence of symptoms 2) doc of hypoglycemia with symptoms and 3) imp of symptoms when glucose raises.

Whipples triad-

An intensive care client with diabetic ketoacidosis (DKA) is receiving an insulin infusion. The cardiac monitor shows ventricular ectopy. Which assessment does the nurse make? Urine output 12-lead electrocardiogram (ECG) Potassium level Correct Rate of IV fluids

With insulin therapy, serum potassium levels fall rapidly as potassium shifts into the cells. Detecting and treating the underlying cause is essential. Insulin treats symptoms of diabetes by putting glucose into the cell as well as potassium; ectopy, indicative of cardiac irritability, is not associated with changes in urine output. A 12-lead ECG can verify the ectopy, but the priority is to detect and fix the underlying cause. Increased fluids treat the symptoms of dehydration secondary to DKA, but do not treat the cause.

Rapid Acting Insulin

With meals Humalong -onset: 15-30 mins -peak: 30-90 mins -duration: 3-5 hours lispro, aspart, glulisine

Short Acting Insulin

With meals Regular -onset: 30min-1 hour -peak: 2-5 hours -duration: 5-8 hours ACHS Type 1 Regular (Humulin R, Novolin R)

liver

Without adequate insulin fat stores break down and increased triglycerides are stored in the

gout

Without adequate insulin, the storage of protein halts and large amounts of amino acids are dumped in the bloodstream increasing the pt's chance of

A patient diagnosed with type 2 diabetes mellitus is admitted to the medical unit with pneumonia. The patient's oral antidiabetic medication has been discontinued and the patient is now receiving insulin for glucose control. Which of the following statements best explains the rationale for this change in medication? Choose 1 answer: A. Stress-related states such as infections increase risk of hyperglycemia B. Infection has compromised beta cell function so the patient will need insulin from now on C. Insulin administration will help prevent hypoglycemia during the illness D. Acute illnesses like pneumonia will cause increased insulin resistance

a

Which of the following laboratory results would alert the healthcare provider that a patient who has diabetes is experiencing diabetic nephropathy? Choose 1 answer: A Microalbuminuria B Hemoglobin A1c 6% C Decreased BUN D Ketonuria

a

The nurse is teaching a client with newly diagnosed type 2 diabetes about the importance of weight control. Which comment by the client indicates a need for further teaching? a. "I will begin exercising for at least an hour a day." b. "I will monitor my diet and avoid empty calories." c. "If I lose weight, I may not need to use the insulin anymore." d. "Weight loss can be a sign of diabetic ketoacidosis."

a Further teaching is needed when the client says that "I will begin exercising for at least an hour a day." The goal of weight control for Type 2 diabetes is to change sedentary behavior to active behavior. This is begun by starting low-intensity activities in short sessions (less than 10 minutes). The client may increase sessions to moderate or vigorous aerobic physical activity to lose and or sustain weight loss. Monitoring the diet and avoiding empty calories is essential to managing type 2 diabetes. Weight loss can minimize the need for insulin and can also be a sign of diabetic ketoacidosis due to osmotic diuresis.

The nurse caring for four clients with diabetes has these activities to perform. Which activity is appropriate to delegate to unlicensed assistive personnel (UAP)? a. Perform a blood glucose check on a client who requires insulin. b. Verify the infusion rate on a continuous infusion insulin pump. c. Assess a client who reports tremors and irritability. d. Monitor a client who is reporting palpitations and anxiety.

a Performing bedside glucose monitoring is a task that may be delegated to UAPs because it does not require extensive clinical judgment to perform. There is no evidence the client is unstable at this time. The nurse will follow up with the results and insulin administration after assessing the less stable clients. Intravenous therapy and medication administration are not within the scope of practice for UAPs. The client with tremors and irritability is displaying symptoms of hypoglycemia requiring further assessment and intervention that are not within the scope of practice for UAPs. The client reporting palpitations and anxiety may have hypoglycemia, requiring further intervention. This client must be assessed by licensed nursing staff.

The nurse working on a medical surgical endocrine unit has just received change-of-shift report. Which client will the nurse see first? a. Client with type 1 diabetes whose insulin pump is beeping "occlusion" b. Newly diagnosed client with type 1 diabetes who is reporting thirst c. Client with type 2 diabetes who has a blood glucose of 150 mg/dL (8.3 mmol/L) d. Client with type 2 diabetes with a blood pressure of 150/90 mm Hg

a The client the nurse sees first is the client with type 1 diabetes whose insulin pump is beeping "occlusion." Because glucose levels will increase quickly in clients whose continuous insulin pumps malfunction, the nurse must assess this client and the insulin pump first to avoid hyperglycemia or diabetic ketoacidosis. Thirst is an expected symptom of hyperglycemia and, although important, is not a priority. The nurse could delegate fingerstick blood glucose to unlicensed assistive personnel while assessing the client whose insulin pump is beeping. Although a blood glucose reading of 150 mg/dL (8.3 mmol/L) is mildly elevated, this does not require immediate action. Mild hypertension does not require immediate action. The nurse can later assess if this is within the client's usual range or represents a change before taking action.

A client with type 1 diabetes arrives in the emergency department breathing deeply and stating, "I can't catch my breath." The client's vital signs are: T 98.4°F (36.9°C), P 112 beats/min, R 38 breaths/min, BP 91/54 mm Hg, and O2 saturation 99% on room air. Which action will the nurse take first? a.Check the blood glucose b.administer oxygen. c.Offer reassurance. d.Attach a cardiac monitor.

a The nurse would first obtain the client's glucose level. Breathing deeply and stating, "I can't catch my breath" is indicative of Kussmaul respirations which is a sign of diabetic ketoacidosis. Based on the oxygen saturation, oxygen administration is not indicated. The nurse provides support, but it is early in the course of assessment and intervention to offer reassurance without more information. Cardiac monitoring may be implemented, but the first action would be to obtain the glucose level.

The patient received 10 units of regular insulin subcutaneously at 8:30 PM for a blood glucose level of 253 mg/dL. You plan to monitor this patient for signs of hypoglycemia at which peak action times? A. 9:00 PM to 10:30 PM B. 10:30 PM to 11:30 PM C. 12:30 AM to 1:30 AM D. 2:30 AM to 4:30 AM

B. 10:30 PM to 11:30 PM Rationale. Regular insulin exerts peak action in 2 to 3 hours making the patient most at risk for hypoglycemia between 10:30 PM and 11:30 PM.

The patient has diabetes mellitus and macroalbuminuria. The patient asks you why the physician is prescribing the angiotensin-converting enzyme (ACE) inhibitor lisinopril (Zestril) for him even though his blood pressure is well-controlled. What is your response? A. It helps prevent hypertension as diabetics are prone to it. B. ACE inhibitors delay the progression of nephropathy in patients with diabetes. C. ACE inhibitors prevent macrovascular complications. D. ACE inhibitors help prevent atherosclerosis.

B. ACE inhibitors delay the progression of nephropathy in patients with diabetes. Rationale. ACE inhibitors and angiotensin II receptor antagonists (ARBs) are used to treat hypertension and delay the progression of nephropathy in patients with diabetes. ACE inhibitors are not used prophylactically. ACE inhibitors do not affect macrovascular complications. Nephropathy is a microvascular complication.

The patient is managed with NPH and regular insulin injections before breakfast and before dinner. When is the patient most likely to have a hypoglycemic reaction? A. After breakfast B. Before lunch C. During lunch D. After lunch

B. Before lunch Rationale. The regular insulin peak occurs about 2 to 3 hours with a duration of 5 to 6 hours. If too much insulin or not enough food is given, the most likely time of hypoglycemia is before lunch, when the regular insulin is still present, the NPH has its onset, and the breakfast food has been metabolized.

What is the proper teaching regarding diabetic foot care? A. Cut toenails so corners are cut down. B. Check your feet daily with a mirror. C. Walk barefoot only in dry summer weather. D. Check the temperature of bath water with your toes.

B. Check your feet daily with a mirror. Rationale. Feet should be inspected daily for broken areas that are at risk for delayed wound healing. Using a mirror allows visualization of areas not readily seen. Toenails should be cut straight across and the corners should not be cut down. Properly fitting shoes should be worn at all times to protect the feet. Improper footwear and stepping on foreign objects when barefoot are common causes of undetected foot injury. Water temperature should be tested with the hands first.

Which is the best patient to share a hospital room with a patient who has diabetes mellitus and is scheduled for surgery? A. Admitted with community-acquired pneumonia B. History of asthma on nebulized breathing treatments C. Cultured vancomycin-resistant enterococcus (VRE) in the stool D. History of tuberculosis and on medications for 1 week

B. History of asthma on nebulized breathing treatments Rationale. The patient with asthma is on nebulized breathing treatments. This patient is not infectious. A patient with diabetes mellitus is more susceptible to infections and should not cohabit with an infectious patient, which all the other patients are.

You are beginning to teach a diabetic patient about the vascular complications of diabetes. Which information is appropriate for you to include? A. Macroangiopathy does not occur in type 1 diabetes but does affect type 2 diabetics who have severe disease. B. Microangiopathy is specific to diabetes and most commonly affects the capillary membranes of the eyes, kidneys, and skin. C. Renal damage resulting from changes in large and medium-sized blood vessels can be prevented by careful glucose control. D. Macroangiopathy causes slowed gastric emptying and the sexual impotency experienced by most patients with diabetes.

B. Microangiopathy is specific to diabetes and most commonly affects the capillary membranes of the eyes, kidneys, and skin. Rationale. Microangiopathy occurs in type 1 and type 2 diabetes mellitus. When it affects the eyes, it is called diabetic retinopathy. When kidneys are affected, the patient has nephropathy. When the skin is affected, it can lead to diabetic foot ulcers. Sexual impotency and slowed gastric emptying result from microangiopathy.

The nurse has just received change-of-shift report on the endocrine unit. Which client does the nurse see first? Client with type 1 diabetes whose insulin pump is beeping "occlusion" Correct Newly diagnosed client with type 1 diabetes who is reporting thirst Client with type 2 diabetes who has a blood glucose of 150 mg/dL Client with type 2 diabetes with a blood pressure of 150/90 mm Hg

Because glucose levels will increase quickly in clients who use continuous insulin pumps, the nurse should assess this client and the insulin pump first to avoid diabetic ketoacidosis. Thirst is a symptom of hyperglycemia and, although important, is not a priority; the nurse could delegate a fingerstick blood glucose to unlicensed assistive personnel while assessing the client whose insulin pump is beeping. Although a blood glucose reading of 150 mg/dL is mildly elevated, this is not an emergency. Mild hypertension is also not an emergency.

A client with type 2 diabetes has been admitted for surgery, and the health care provider has placed the client on insulin in addition to the current dose of metformin (Glucophage). The client wants to know the purpose of taking the insulin. What is the nurse's best response? "Your diabetes is worse, so you will need to take insulin." "You can't take your metformin while in the hospital." "Your body is under more stress, so you'll need insulin to support your medication." Correct "You must take insulin from now on because the surgery will affect your diabetes."

Because of the stress of surgery and NPO status, short-term insulin therapy may be needed perioperatively for the client who uses oral antidiabetic agents. For those receiving insulin, dosage adjustments may be required until the stress of surgery subsides. No evidence suggests that the client's diabetes has worsened; however, surgery is stressful and may increase insulin requirements. Metformin may be taken in the hospital; however, not on days when the client is NPO for surgery. When the client returns to his or her previous health state, oral agents will be resumed.

high protein diets until they went into acidosis and died

Before insulin was developed, people with diabetes were treated with

Metformin

Biguanide (only type used in US) Glucophage, Glucophage XR Reduces glucose produced by the liver

Dopamine Receptor Agonists

Bromocriptien Stimulate dopamine receptors in brain SE: GI effects, dizziness, weakness Activates dopamine receptors in central nervous system. Unknown how it improves glycemic levels.

postural hypotension, resting tachy, exercise intolerance, and failure of the heart rate to rise with exercise

Cardiovascular system symptoms related to autonomic neuropathy

What is the best teaching for a patient who is newly diagnosed with diabetes mellitus type 2? A. Read a Snellen chart yearly. B. Be checked out for presbycusis. C. Notify the doctor if your vision has color distortion. D. See an ophthalmologist for a dilated eye examination yearly.

D. See an ophthalmologist for a dilated eye examination yearly. Rationale. The earliest and most treatable stages of diabetic retinopathy often produce no changes in the vision. Because of this, the patient with type 2 diabetes should have a dilated eye examination by an ophthalmologist at the time of diagnosis and annually thereafter for early detection and treatment. The Snellen test is used to determine vision acuity and the need for glasses, and it is not specific to diabetes. Presbycusis is age-related hearing impairment and is not specific to diabetes. Cataracts related to age typically cause some vision distortion.

What is most helpful in the prevention of nephropathy in a patient with diabetes mellitus? A. Acid-ash diet B. Ensuring adequate fluid intake for renal perfusion C. Preventing obstruction from benign prostatic hyperplasia (BPH) D. Stopping smoking

D. Stopping smoking Rationale. Risk factors for diabetic nephropathy include hypertension, genetic predisposition, smoking, and chronic hyperglycemia. Diabetic nephropathy is an intrarenal microvascular complication in which the glomeruli of the kidney are damaged. The acid-ash diet is used to prevent kidney stones, a potential postrenal cause. The kidney receives about 25% of the cardiac output, and inadequate fluids or shock resulting in adequate perfusion is a prerenal cause. BPH is a postrenal cause of kidney pathology.

Which skin condition should you routinely assess in a patient with diabetes mellitus? A. Contact dermatitis B. Psoriasis C. Malignant melanoma D. Tinea pedis

D. Tinea pedis Rationale. A patient with diabetes is more susceptible to infections than other patients. The dark, moist environment of the feet make the diabetic patient more prone to athlete's foot, a fungal infection. Contact dermatitis is an inflammatory response after contact with a specific antigen and is not related to diabetes. Psoriasis is a chronic, noninfectious skin inflammation involving keratin synthesis and is not related to diabetes. Malignant melanoma is a skin cancer that can metastasize. Although important to notice, infectious skin conditions are more commonly related to diabetes mellitus.

24

DKA accounts for half the deaths off diabetic pt's younger than

diabetes isn't dx'd, pt doesn't take enough insulin, pt with type 1 diabetes exercises too vigorously, or when the pt experiences stress linked to illness, inf, surgery, or emotions.

DKA is likely to occur when

increase in glucose levels between 5 and 9 am not related to hypoglycemia

Dawn phenomenom

Sodium Glucose Co- Transporter 2 Inhibitors

Decreases renal glucose reabsorption and increases urinary glucose excretion.

diabetic retinopathy, & nephropathy

Diabetes is the most common cause of

The nurse is teaching a client about the manifestations and emergency treatment of hypoglycemia. In assessing the client's knowledge, the nurse asks the client what he or she should do if feeling hungry and shaky. Which response by the client indicates a correct understanding of hypoglycemia management? "I should drink a glass of water." "I should eat three graham crackers." Correct "I should give myself 1 mg of glucagon." "I should sit down and rest."

Eating three graham crackers is a correct management strategy for mild hypoglycemia. Water or resting does not remedy hypoglycemia. Glucagon should be administered only in cases of severe hypoglycemia.

Dipeptidyl Peptidase-4 (DPP-4) Inhibitors

Enhances activity of incretins. Stimulate release of insulin from pancreatic B-cells. Decrease hepatic glucose production.

cataract, retinopathy, and glaucoma

Eye changes that pt's with DM are most succeptible

ask if they've had floaters, diplopia, or blurred vision and halos.

Eye changes to assess in pt

too much insulin, not eating, gastroperesis, renal insufficiency, and certain drugs such as asa, and beta blockers.

Factors that can trigger hypoglycemia

poor glucose control, htn, long standing diabetes, and genetic susceptibility

Factors that contribute to the dev of nephropathy

What therapies are appropriate for patients with diabetes mellitus (select all that apply)? A. Use of statins to treat dyslipidemia B. Use of diuretics to treat nephropathy C. Use of angiotensin-converting enzyme (ACE) inhibitors to treat nephropathy D. Use of laser photocoagulation to treat retinopathy E. Use of protein restriction in patients with early signs of nephropathy

A. Use of statins to treat dyslipidemia C. Use of angiotensin-converting enzyme (ACE) inhibitors to treat nephropathy D. Use of laser photocoagulation to treat retinopathy Rationale. In patients with diabetes who have microalbuminuria or macroalbuminuria, ACE inhibitors (e.g., lisinopril [Prinivil, Zestril]) or angiotensin II receptor antagonists (ARBs) (e.g., losartan [Cozaar]) should be used. Both of these drug classes are used to treat hypertension and have been found to delay the progression of nephropathy in patients with diabetes. The statin drugs are the most widely used lipid-lowering drugs. Laser photocoagulation therapy is indicated to reduce the risk of vision loss in patients with proliferative retinopathy, macular edema, and in some cases of nonproliferative retinopathy.

The patient presents to the emergency department with a glucose level of 400 mg/dL, ketone result of 2+, and rapid respirations with a fruity odor. What finding do you anticipate? A. pH below 7.30 B. Urine specific gravity below 1.005 C. High sodium bicarbonate levels D. Low blood urea nitrogen (BUN) level

A. pH below 7.30 Rationale. The patient is in metabolic acidosis, which is a pH below 7.35. Dehydration results in a high urine specific gravity (at the upper end of the normal range, or above 1.025 to 1.030). Sodium bicarbonate levels are low in metabolic acidosis. The dehydration that occurs with DKA elevates the BUN level.

2-3 months

A1C is an accumulation of how long?

Preferred site for injections

Abdomen

accumulates ketones as a result of breaking down fat for energy b/c of inadequate insulin.

Acidosis in type 1 occurs when the body

shaky, nervous, irritable, anxiety, hunger, tachy, tigling and numbness of the lips or tongue, and diaphoresis

Adrenergic symptoms of hypoglycemia-

polydipsia, polyuria, and poly phagia

As DKA progresses the symptoms are

onion, garlic, dandelion, and ginseng

Herbal supplements that can lower glucose

Thiazolidinediones

Increase peripheral muscle glucose uptake and decrease glucose output from the liver. Pioglitazone Rosiglitazone sensitizes peripheral tissues to insulin rosiglitazone - avandia pioglitazone - actos

decr LOC, polyuria,& polydipsia in the absence of ketosis

Indications of HHNS

Long Acting Insulin

Insulin Glargine (Lantus); O: 1.1 hr. P: 14-20 hrs. (Don't Mix)

fatty acids and proteins

Insulin is not only needed for glucose metabolism but the synthesis of

fatty acids into fat,& enhances protein synthesis

Insulin promotes

endothelial tissue (the lining of the heart, blood, and lymph vessels

It is believed that metabolic syndrome represents a chronic low grade inflammatory process affecting

severe hyperglycemia associated with another acute cond such as sepsis or a MI

Ketoacidosis in type 2 diabetes occurs with

slow infusion of insulin, once the serum level reaches 250-300 dextrose solution is added.

Ketoacidosis is treated with

kussmauls respirations, coma, and shock

Late signs of DKA

Insulin glargine (Lantus) and Insulin detemir (Levemir)

Long acting insulin ex-

microvascular, macrovascular, and neuropathic

Long term complications can be classified as

atherosclerosis, ischemic heart disease, left ventricular hypertrophy, and sometimes type 2 dm

Long term effects of metabolic syndrome

Sulfonylureas

MOA: Increase pancreatic insulin secretion ("secretagogues") oral antidiabetic agents used to stimulate the pancreas to release more insulin glyburide and glipizide Can cause hypoglycemia

It causes the loss of central vision

Macular edema occurs in both types of diabetic retinopathy, which affects which part of the vision?

caused by rough shoe lining, or attempts at shaving or clipping nails

Mechanical irritation that causes foot ulcers are

The patient with type 1 diabetes arrives in the emergency department with a glucose level of 390 mg/dL and positive result for ketones. Vital signs are 110/70 mm Hg, 120 beats/minute, and 28 deep, sighing respirations/minute. What is the priority need for the patient? A. Oxygen B. Intravenous (IV) fluids C. Albuterol (Ventolin) D. Metformin (Glucophage)

B Intravenous (IV) Fluids Rationale. A patient in diabetic ketoacidosis (DKA) needs IV fluids and insulin to stop the tissue breakdown resulting in ketone bodies and acidosis. The initial goal is fluid and electrolyte balance. Kussmaul respirations indicate the body is attempting to compensate by blowing off the carbon dioxide, but it is ineffective as long as the body continues to break down the ketone bodies and remains in metabolic acidosis. The issue is not respiratory insufficiency, and a bronchodilator is not indicated. Diabetic ketoacidosis occurs in type 1 diabetes and requires insulin; the pancreas no longer has the ability to respond to oral hypoglycemic medication.

The patient with type 1 diabetes mellitus eats a large meal but does not take his insulin as prescribed. In what order do the following physiologic events occur in the development of a hyperglycemic emergency? (Answer with a letter followed by a comma and a space (e.g. A, B, C, D).) _________ a. Fat metabolism leads to ketonemia. b. Insufficient insulin in the blood stream c. Fat is mobilized for energy from the adipose tissue. d. Potassium and water are excreted with H+ and ketones in the urine. e. Organic acid accumulation in the blood causes metabolic acidosis.

B, C, A, E, D A patient with type 1 diabetes who develops blood glucose greater than 250 mg/dL is at risk to develop diabetic ketoacidosis (DKA). In DKA, the patient's cells lack fuel (glucose), despite excess serum glucose because there is insufficient insulin available to metabolize the glucose and drive it into the cells. So the body turns to fat for fuel and produces ketones as a byproduct of fat metabolism. Ketonemia decreases the serum pH as H+ ions accumulate, and hyperglycemia causes osmotic diuresis, leading to the excretion of electrolytes, especially potassium and chloride (as a carrier of H+).

The nurse is teaching a 60-year-old woman with type 2 diabetes mellitus how to prevent diabetic nephropathy. Which statement made by the patient indicates that teaching has been successful? A. "Smokeless tobacco products decrease the risk of kidney damage." B. "I can help control my blood pressure by avoiding foods high in salt." C. "I should have yearly dilated eye examinations by an ophthalmologist." D. "I will avoid hypoglycemia by keeping my blood sugar above 180 mg/dL."

B. "I can help control my blood pressure by avoiding foods high in salt." Rationale. Diabetic nephropathy is a microvascular complication associated with damage to the small blood vessels that supply the glomeruli of the kidney. Risk factors for the development of diabetic nephropathy include hypertension, genetic predisposition, smoking, and chronic hyperglycemia. Patients with diabetes are screened for nephropathy annually with a measurement of the albumin-to-creatinine ratio in urine; a serum creatinine is also needed.

A client with type 1 diabetes mellitus received regular insulin at 7:00 a.m. The client should be monitored for hypoglycemia at which time? 7:30 a.m. 11:00 a.m. Correct 2:00 p.m. 7:30 p.m.

Onset of regular insulin is ½ to 1 hour; peak is 2 to 4 hours. Therefore, 11:00 a.m. is the anticipated peak time for regular insulin received at 7:00 a.m. For regular insulin received at 7:00 a.m., 7:30 a.m., 2:00 p.m., and 7:30 p.m. are not the anticipated peak times.

15-30 min onset, 30-90 min peak, and 3-5 hour duration.

Onset, peak, and duration of rapid acting-

between 70 and 130 before meals, and less than 180 after meals

Optimal glucose levels for diabetics are

sulfonylureas, alpha glucosidase inhibitors, biguanides, thiazolidinediones, meglitinides, and other inhibitors

Oral hypoglycemic include

The nurse is providing discharge teaching to a client with type 2 diabetes and peripheral neuropathy. Which statement by the client indicates a need for further teaching about injury prevention? a. "I can break in my shoes by wearing them all day." b. "I need to monitor my feet daily for blisters or skin breaks." c. "I will never go barefoot." d. "I need to quit smoking."

a further teaching about injury prevention is needed when the client with diabetic peripheral neuropathy says that "I can break in my shoes by wearing them all day." Shoes need to be properly fitted and worn for a few hours a day to break them in, with frequent inspection for irritation or blistering. People with diabetes have decreased peripheral circulation, so even small injuries to the feet must be managed early. Going barefoot is contraindicated because if the client has diabetic neuropathy, stepping on something sharp or harmful would not be felt. Tobacco use further decreases peripheral circulation increasing the risk for vascular complications.

Which is the best referral that the community health nurse can suggest to a client who has been newly diagnosed with diabetes? a. American Diabetes Association (ADA) b. Centers for Disease Control and Prevention c. Primary health care provider office d. Pharmaceutical representative

a he American Diabetes Association is the best agency to refer the diabetic client to. The ADA provides national and regional support and resources to clients with diabetes and their families. The Centers for Disease Control and Prevention does not specifically focus on diabetes. The client's primary health care provider's office is limited in the resources available to the client with diabetes. A pharmaceutical representative is not an appropriate resource for diabetes information and support.

Gestational Diabetes

a form of diabetes mellitus that occurs during some pregnancies abnormally high blood-sugar levels in the mother

Ketoacidosis

a state due to lack of insulin in which excess ketones and excess acids are produced abnormal condition of ketones in the blood as a result of diabetes mellitus

A client with type 1 diabetes arrives in the emergency department breathing deeply and stating, "I can't catch my breath." The client's vital signs are: T 98.4°F (36.9°C), P 112 beats/min, R 38 breaths/min, BP 91/54 mm Hg, and O2 saturation 99% on room air. Which action will the nurse take first? a. Check the blood glucose. b. Administer oxygen. c. Offer reassurance. d. Attach a cardiac monitor.

a. Check the blood glucose

The nurse working on a medical surgical endocrine unit has just received change-of-shift report. Which client will the nurse see first? a. Client with type 1 diabetes whose insulin pump is beeping "occlusion" b. Newly diagnosed client with type 1 diabetes who is reporting thirst c. Client with type 2 diabetes who has a blood glucose of 150 mg/dL (8.3 mmol/L) d. Client with type 2 diabetes with a blood pressure of 150/90 mm Hg

a. Client with type 1 diabetes whose insulin pump is beeping "occlusion"

The nurse is providing discharge teaching to a client with type 2 diabetes and peripheral neuropathy. Which statement by the client indicates a need for further teaching about injury prevention? a. "I can break in my shoes by wearing them all day." b. "I need to monitor my feet daily for blisters or skin breaks." c. "I will never go barefoot." d. "I need to quit smoking."

a. I can break in my shoes by wearing them all day

The nurse is teaching a client with newly diagnosed type 2 diabetes about the importance of weight control. Which comment by the client indicates a need for further teaching? a. "I will begin exercising for at least an hour a day." b. "I will monitor my diet and avoid empty calories." c. "If I lose weight, I may not need to use the insulin anymore." d. "Weight loss can be a sign of diabetic ketoacidosis."

a. I will begin exercising for at least an hour a day

The nurse caring for four clients with diabetes has these activities to perform. Which activity is appropriate to delegate to unlicensed assistive personnel (UAP)? a. Perform a blood glucose check on a client who requires insulin. b. Verify the infusion rate on a continuous infusion insulin pump. c. Assess a client who reports tremors and irritability. d. Monitor a client who is reporting palpitations and anxiety.

a. Perform a blood glucose check on a client who requires insulin

The healthcare provider is assessing the glucose level of a patient with a diagnosis of diabetes. Which of these is most helpful in evaluating this patient's long-term glucose management? Choose 1 answer: A. Fasting blood glucose level B. Hemoglobin A1c C. Urine specific gravity D. The patient's food diary

b

The healthcare provider is teaching a group of students about the characteristics of type 1 diabetes mellitus. Which of the following describe the underlying cause of the disease? Choose 1 answer: A. Atrophy of pancreatic alpha cells B. Destruction of pancreatic beta cells C. Cellular resistance to insulin D. Increased hepatic glycogenesis

b

The nurse is teaching a client about the manifestations and emergency management of hypoglycemia. Which response by the client indicates a correct understanding of what to do if the client feels hungry and shaky? a. "I will drink a glass of water." b. "I will eat three graham crackers." c. "I will give myself 1 mg of glucagon." d. "I will sit down and rest."

b Correct understanding of what the client needs to do if the client feels hungry and shaky is to eat three graham crackers. This is the correct management strategy for mild hypoglycemia. Drinking a glass of water or sitting down and resting does not remedy hypoglycemia. Glucagon is generally administered for episodes of severe not mild hypoglycemia.

The nurse in the endocrine clinic is providing education for a client who has just been diagnosed with diabetes. Which factor is most important for the nurse to assess before providing instruction to the client about the disease and its management? a. Current lifestyle b. Educational and literacy level c. Sexual orientation d. Current energy level

b The most important factor for the nurse to determine before providing instruction to the newly diagnosed client with diabetes is the client's educational level and literacy level. A large amount of information must be synthesized. Written instructions are typically given. The client's ability to learn and read is essential to provide the client with instructions and information about diabetes. Although lifestyle would be taken into account, it is not the priority. Sexual orientation will have no bearing on the ability of the client to provide self-care. Although energy level will influence the ability to exercise, it is not essential.

The nurse has just taken change-of-shift report on a group of clients on the medical-surgical unit. Which client does the nurse assess first? a. Client taking repaglinide (Prandin) who has nausea and back pain b. Client taking glyburide (Diabeta) who is dizzy and sweaty c. Client taking metformin (Glucophage) who has abdominal cramps d. Client taking pioglitazone (Actos) who has bilateral ankle swelling

b The nurse needs to first assess the client taking glyburide (Diabeta) who is dizzy and sweaty and has symptoms consistent with hypoglycemia. Because hypoglycemia is the most serious adverse effect of antidiabetic medications, this client must be assessed as soon as possible. Nausea is a documented side effect of repaglinide. Checking the client's back pain requires assessment, which can be performed after the nurse assesses the client displaying signs and symptoms of hypoglycemia. Metformin may cause abdominal cramping and diarrhea, but the client taking it does not require immediate assessment. Ankle swelling is an expected side effect of pioglitazone.

The nurse is teaching a client with diabetes about proper foot care. Which statement by the client indicates that teaching was effective? A. "I will go barefoot in my house so that my feet are exposed to air." B. "I must inspect my shoes for foreign objects before putting them on." C. "I will soak my feet in warm water to soften calluses before trying to remove them." D. "I must wear canvas shoes as much as possible to decrease pressure on my feet."

b The statement by the diabetic client that indicates that teaching was effective is, "I must inspect my shoes for foreign objects before putting them on." To avoid injury or trauma to the feet, shoes need to be checked for foreign objects before the feet are inserted in them. Clients with diabetes would not go barefoot because foot injuries can occur in those clients who lack sensation. To avoid injury or trauma, a callus needs to be removed by a podiatrist, not by the client. To prevent injury, the client with diabetes must wear protective shoes for support and not canvas shoes.

The clinic nurse is providing teaching to a client with newly diagnosed diabetes. Which statement by the client indicates a correct understanding about the need to wear a MedicAlert bracelet? a. "If I become hyperglycemic, it is a medical emergency." b. "If I become hypoglycemic, I could become unconscious." c. "Medical personnel may need confirmation of my insurance." d. "I may need to be admitted to the hospital suddenly."

b The statement by the client that indicates a correct understanding about the need to wear a MedicAlert bracelet is, "If I become hypoglycemic, I could become unconscious." Hypoglycemia is the most common cause of medical emergency in clients with diabetes. A MedicAlert bracelet is helpful if the client becomes hypoglycemic and is unable to provide self-care. Hyperglycemia does not pose the same type of acute medical emergency as hypoglycemia unless it is severe and acidosis develops. Insurance information does not appear on a MedicAlert bracelet. Information on the MedicAlert bracelet may be helpful if a sudden hospitalization occurs when the client cannot communicate. However, it is standard procedure to assess blood glucose in that instance.

A client newly diagnosed with diabetes is not ready to learn diabetes control during the hospital stay. Which information is the priority for the nurse to teach the client and the client's family? Select all that apply. a. Pathophysiology of diabetes b. Causes and treatment of hypoglycemia c. Dietary control of blood glucose d. Insulin administration e. Physical activity and exercise

b, d

A client newly diagnosed with diabetes is not ready to learn diabetes control during the hospital stay. Which information is the priority for the nurse to teach the client and the client's family? (Select all that apply.) A. Pathophysiology of diabetes B. Causes and treatment of hypoglycemia C. Dietary control of blood glucose D. Insulin administration E. Physical activity and exercise

b, d The priority information the nurse needs to teach the client and family about diabetes are the causes and treatment of hypoglycemia and proper insulin administration. This information is essential for the client's survival and must be understood by both the client and family to ensure client safety. The pathophysiology of diabetes and hyperglycemia is a topic for secondary teaching and is not a survival need or the priority during hospitalization. Dietary control and exercise regimen are important, but are not the priority during the acute care stay.

A client with type 1 diabetes mellitus received regular insulin at 7:00 a.m. The client will need to be monitored for hypoglycemia at which time? a. 7:30 a.m. b. 11:00 a.m. c. 2:00 p.m. d. 7:30 p.m.

b. 11am

The nurse in the endocrine clinic is providing education for a client who has just been diagnosed with diabetes. Which factor is most important for the nurse to assess before providing instruction to the client about the disease and its management? a. Current lifestyle b. Educational and literacy level c. Sexual orientation d. Current energy level

b. Educational and literacy level

The nurse is teaching a client with diabetes about proper foot care. Which statement by the client indicates that teaching was effective? a. "I will go barefoot in my house so that my feet are exposed to air." b. "I must inspect my shoes for foreign objects before putting them on." c. "I will soak my feet in warm water to soften calluses before trying to remove them." d. "I must wear canvas shoes as much as possible to decrease pressure on my feet."

b. I must inspect my shoes for foreign objects before putting them on

The nurse is teaching a client about the manifestations and emergency management of hypoglycemia. Which response by the client indicates a correct understanding of what to do if the client feels hungry and shaky? a. "I will drink a glass of water." b. "I will eat three graham crackers." c. "I will give myself 1 mg of glucagon." d. "I will sit down and rest."

b. I will eat three graham crackers

The clinic nurse is providing teaching to a client with newly diagnosed diabetes. Which statement by the client indicates a correct understanding about the need to wear a MedicAlert bracelet? a. "If I become hyperglycemic, it is a medical emergency." b. "If I become hypoglycemic, I could become unconscious." c. "Medical personnel may need confirmation of my insurance." d. "I may need to be admitted to the hospital suddenly."

b. If I become hypoglycemic, I could become unconscious

The nurse has just taken change-of-shift report on a group of clients on the medical-surgical unit. Which client does the nurse assess first? a. Client taking repaglinide (Prandin) who has nausea and back pain b. Client taking glyburide (Diabeta) who is dizzy and sweaty c. Client taking metformin (Glucophage) who has abdominal cramps d. Client taking pioglitazone (Actos) who has bilateral ankle swelling

b. client taking glyburide (Diabetes) who is dizzy and sweaty

A client expresses fear and anxiety over the life changes associated with diabetes, stating, "I am scared I can't do it all and I will get sick and be a burden on my family." What is the nurse's best response? a. "It is overwhelming, isn't it?" b. "Let's see how much you can learn today, so you are less nervous." c. "Let's tackle it piece by piece. What is most scary to you?" d. "Many people live with diabetes and do it just fine."

c The nurse's best response is to suggest that the client tackle it piece by piece and ask what is most scary to him or her. This is the best client centered response, and acknowledges the client's concern, letting the client master survival skills first. Referring to the illness as overwhelming may reflect the client's feelings, but is a closed-ended question and does not encourage the client to express his feelings about the underlying fear. Trying to see how much the client can learn in one day may add to his anxiety by overwhelming him with information and the need to "do it all" in one day. Suggesting that other people handle the illness just fine criticizes the client and does not recognize his concerns.

A diabetic patient has been prescribed an alpha-1 glucosidase inhibitor. When teaching the patient about the medication, which of the following information will the healthcare provider include? Choose 1 answer: A "Call us immediately if you experience tremors or palpitations." B "Take one tablet daily first thing in the morning." C "Take this medication with the first bite of each meal." D "You should select foods low in protein when taking this medication."

c hint: Glucosidase is a hormone that breaks down glucose so it can be absorbed.

A client with typically well controlled diabetes has a glycosylated hemoglobin (HbA1C) level of 9.4%. Which response by the nurse is most appropriate? a. "Keep up the good work." b. "This is not good at all." c. "Have you been doing something differently? d. "You need an increase in your insulin dose."

c. Have you been doing something differently?

A client expresses fear and anxiety over the life changes associated with diabetes, stating, "I am scared I can't do it all and I will get sick and be a burden on my family." What is the nurse's best response? a. "It is overwhelming, isn't it?" b. "Let's see how much you can learn today, so you are less nervous." c. "Let's tackle it piece by piece. What is most scary to you?" d. "Many people live with diabetes and do it just fine."

c. Let's tackle piece by piece. What is most scary to you?

An intensive care client with diabetic ketoacidosis (DKA) is receiving an insulin infusion. When the cardiac monitor shows ventricular ectopy, which assessment will the nurse make? a. Urine output b. 12-lead electrocardiogram (ECG) c. Potassium level d. Rate of IV fluids

c. Potassium level

A client with type 2 diabetes controlled with Metformin is recovering from surgery. The primary health care provider has placed the client on insulin in addition to the metformin. What is the nurse's best response about why the client needs to take insulin? a. "Your diabetes is getting worse, so you will need to take insulin." b. "You can't take your metformin while in the hospital." c. Stress, such as surgery, increases blood glucose levels. You'll need insulin to control your blood glucose temporarily." d. "You must take insulin from now on because the surgery will affect your diabetes."

c. Stress, such as surgery, increases blood glucose levels. You'll need insulin to control your blood glucose temporarily

The nurse in the endocrine clinic is reviewing type 1 and type 2 diabetes with a group of nursing students. Which explanation by the students indicates their understanding of the types of diabetes? a. Most clients with type 1 diabetes are born with it. b. People with type 1 diabetes are often obese. c. Those with type 2 diabetes make insulin, but in inadequate amounts. d. People with type 2 diabetes do not develop typical diabetic complications.

c. Those with type 2 diabetes make insulin, but in inadequate amounts

Which of these clients with diabetes will the endocrine unit charge nurse assign to an RN who has floated from the labor/delivery unit? a. A client with sensory neuropathy who needs teaching about foot care b. A client with diabetic ketoacidosis who has an IV running at 250 mL/hr c. A client who needs blood glucose monitoring and insulin before each meal d. A client who was admitted with fatigue and shortness of breath

c. a client who needs blood glucose monitoring and insulin before each meal

intermediate acting insulin

cloudy insulins-

An unresponsive patient who has diabetes is brought to the emergency department with slow, deep respirations. Additional findings include: blood glucose 450 mg/dL (24.9 mmol/L), arterial pH 7.2, and urinalysis showing presence of ketones and glucose. Which of the following statements best describes the the underlying cause of this patient's presentation? Choose 1 answer: A Hypoglycemia causes release of glucagon resulting in glycogenolysis and hyperglycemia B Nocturnal elevation of growth hormone results in hyperglycemia in the morning C Hyperglycemia causes oxidative stress, renal dysfunction, and acidosis D Lack of insulin causes increased counterregulatory hormones and fatty acid release

d

The healthcare provider administers NPH insulin to a patient who has diabetes at 6:00 AM. When will the patient be at highest risk of experiencing hypoglycemia? Choose 1 answer: A 7:00 AM B 7:30 AM C 9:00 AM D 10:00 AM

d

he healthcare provider is caring for a patient who has diabetes and is also diagnosed with hypertension. Which of the following medications on the patient's medication administration record will cause the most concern? Choose 1 answer: A Angiotensin receptor blocker B ACE inhibitor C Calcium channel blocker D Beta-blocker

d

The intensive care nurse is caring for a client admitted in a hyperglycemic-hyperosmolar state. Which of these prescriptions made by the primary health care provider will the nurse question? a. Add 20 mEq of KCl to each liter of IV fluid b. IV regular insulin at 2 units/hr c. IV normal saline at 100 mL/hr d. 1 ampule Sodium Bicarbonate IV now

d Sodium Bicarbonate is given for the acid-base imbalance of diabetic ketoacidosis, not the hyperglycemic-hyperosmolar state that presents with hyperglycemia and absence of ketosis/acidosis. Insulin puts potassium into the cell. KCl 20 mEq for each liter of IV fluid will correct hypokalemia from osmotic diuresis and electrolyte shifts. IV regular insulin at 2 units/hr will help correct hyperglycemia. IV normal saline at 100 mL/hr will help correct dehydration.

A client recently admitted with new-onset type 2 diabetes will be discharged with a meter for self-monitoring of blood glucose (SMBG) levels. When is the best time for the nurse to explain to the client the proper use of the glucose monitor? a. Day of discharge b. On admission c. When the client states readiness d. While performing the test in the hospital

d Teaching the client about the operation of the machine while performing the test in the hospital is the best time for the nurse to introduce the client to SMBG. The teaching can be reinforced each time testing is performed on the client and again before discharge. Instructing the client on the day of admission or the day of discharge would not allow time for redemonstration and correction of the skill if needed. Other time-consuming activities are done on those days and could distract the client and make the client feel overwhelmed. Also, waiting for the client to state readiness may postpone the instructions too long.

A client with type 2 diabetes who is taking metformin (Glucophage) is seen in the diabetic clinic. The fasting blood glucose is 108 mg/dL (6.0 mmol/L), and the glycosylated hemoglobin (HbA1C) is 8.2%. Which action will the nurse take next? a. Instruct the client to continue with the current diet and metformin use. b. Discuss the need to check blood glucose several times every day. c. Talk about the possibility of adding rapid-acting insulin to the regimen. d. Ask the client about current dietary intake and medication use.

d The nurse's next action would be to assess the client's adherence to the currently prescribed diet and medications. The nurse would also check for any stressors or undocumented illnesses. Glycosylated hemoglobin (HbA1C) levels >8% indicate poor diabetes control and need for adherence to regimen or changes in therapy. Instructing the client to continue with current diet and metformin use is inappropriate without further assessment. Checking blood glucose more frequently and/or using rapid-acting insulin may be appropriate, but this will depend on the assessment data. The HbA1C indicates that the client's average glucose level is higher than the target range, but discussing the need to check blood glucose several times every day assumes that the client is not compliant with the therapy and glucose monitoring. The nurse would not assume that adding insulin, which must be prescribed by the primary health care provider, is the answer without assessing the underlying reason for the treatment failure.

Which nursing action will the home health nurse delegate to a home health aide who is making daily visits to a client with newly diagnosed type 2 diabetes? a. Assist the client's spouse in choosing appropriate dietary items. b. Evaluate the client's use of a home blood glucose monitor. c. Inspect the extremities for evidence of poor circulation. d. Assist the client with washing the feet and applying moisturizing lotion.

d The nursing action that the home health nurse can delegate to a home health aide who is making daily visits to a newly diagnosed type 2 diabetic client is assisting with personal hygiene. This action is included in the role of home health aides. Assisting with appropriate dietary selections, evaluating the effectiveness of teaching, and performing assessments are complex actions that would be performed by licensed nurses.

The intensive care nurse is caring for a client admitted in a hyperglycemic-hyperosmolar state. Which of these prescriptions made by the primary health care provider will the nurse question? a. Add 20 mEq of KCl to each liter of IV fluid b. IV regular insulin at 2 units/hr c. IV normal saline at 100 mL/hr d. 1 ampule Sodium Bicarbonate IV now

d. 1 ampule sodium bicarbonate IV now

A client with type 2 diabetes who is taking metformin (Glucophage) is seen in the diabetic clinic. The fasting blood glucose is 108 mg/dL (6.0 mmol/L), and the glycosylated hemoglobin (HbA1C) is 8.2%. Which action will the nurse take next? a. Instruct the client to continue with the current diet and metformin use. b. Discuss the need to check blood glucose several times every day. c. Talk about the possibility of adding rapid-acting insulin to the regimen. d. Ask the client about current dietary intake and medication use.

d. Ask the client about current dietary intake and medication use

Which nursing action will the home health nurse delegate to a home health aide who is making daily visits to a client with newly diagnosed type 2 diabetes? a. Assist the client's spouse in choosing appropriate dietary items. b. Evaluate the client's use of a home blood glucose monitor. c. Inspect the extremities for evidence of poor circulation. d. Assist the client with washing the feet and applying moisturizing lotion.

d. Assist the client with washing the feet and applying moisturizing

The nurse is performing an admission assessment on a 52-year-old client admitted with type 2 diabetes.Physical Assessment Diagnostic Findings Provider Prescriptions Lungs clear Glucose 179 mg/dL (9.9 mmol/L)Regular insulin 8 units if blood glucose 250 to 275 mg/dL (13.9 to 15.3 mmol/L)Right great toe mottled and cold to touch Hemoglobin A1c 6.9% Regular insulin 10 units if glucose 275 to 300 mg/dL (15.3 to 16.7 mmol/L)Client states wears eyeglasses to read After completing the above assessment, which complication of diabetes does the nurse report to the primary health care provider? a. Poor glucose control b. Visual changes c. Respiratory distress d. Decreased peripheral perfusion

d. Decreased peripheral perfusion

A client recently admitted with new-onset type 2 diabetes will be discharged with a meter for self-monitoring of blood glucose (SMBG) levels. When is the best time for the nurse to explain to the client the proper use of the glucose monitor? a. Day of discharge b. On admission c. When the client states readiness d. While performing the test in the hospital

d. While performing the test in the hospital

Type 2 Diabetes Mellitus

disorder of insulin recognition by cells, adult onset, obesity is a major risk factor a form of diabetes in which the pancereas produces some insulin that doesnt do its job Represents approximately 90% of the cases of D.M. non insulin dependent

corticosteroids, sympathomimetics, and thiazide diuretics

drugs that raise serum levels

related to tumors or genetics

endogenous causes of hypoglycemia

produced by the body

endogenous insulin

4-6 oz of apple, orange juice, and soda, 8 oz skim milk, 1 tsb sugar, 1 tbs syrup or honey, 6-8 life savers, 3-4 glucose tabs, and 15 g glucose gel

ex of quick acting carbs

insuin, oral hypoglycemic, alcohol, and exercise

exogenous causes of hypoglycemia

gastric surgery, fasting, and malnutrition

functional causes of hypoglycemia

when the stomach dilates and loses muscle tone causing gastric emptying to be delayed

gastroparesis is related to autonomic neuropathy and is

Types of sulfonylureas

glipizide (Glucotrol) glimepiride (Amaryl)

the way the body reacts to carb ingestion with insulin

glycogenesis

Purpose for sulfonylureas

increase insulin production from the pancreas

Diabetes Mellitus

insulin is not secreted adequately or tissues are resistant to its effects Metabolic disorder characterized by high blood glucose and insufficient insulin

Long-Acting (basal) time frame of injection, onset, peak, duration?

-Injected once QD HS or in the AM Onset: 1-2 hours No peak action Duration: 24 hours

The 4 methods of diagnostic studies

1. A1C GREATER THAN OR EQUAL TO 6.5% 2. Fasting Plasma Glucose (FPG) greater than or equal to 126mg/dL (7.0mmol/L) 3. Two hour plasma glucose level greater than or equal to 200 mg/dl (11.0mmol/L) using glucose load of 75g during an OGTT 4. Random Plasma Glucose greater than or equal to 200mg/dL (11.0mmol/L) with symptoms of hyperglycemia

Impaired fasting glucose (IFG) range

100-125 mg/dL

Impaired glucose tolerance (IGT) range

140-199mg/dL

Analyze the following diagnostic finding for your patient with type 2 diabetes. Which result will need further assessment? A. A1C 9% B. BP 126/80 mmHg C. FBG 130mg/dL (7.2 mmol/L) D. LDL cholesterol 100 mg/dL (2.6mmol/dL)

A. A1C 9% Rationale. Lowering hemoglobin A1C (to less than 7%) reduces microvascular and neuropathic complications. Tighter glycemic control (normal hemoglobin A1C level, less than 6%) may further reduce complications but increases hypoglycemia risk.

What is the correct teaching regarding oral antidiabetic medications? A. Double the glipizide (Glucotrol) dose if consuming alcohol. B. Hold all antidiabetic medication if vomiting is related to the flu. C. Hold metformin 48 hours before a procedure with contrast medium. D. Acarbose (Precose) is taken 2 hours after meals.

C. Hold metformin 48 hours before a procedure with contrast medium. Rationale. Metformin (Glucophage) is held 24 to 48 hours before and after a procedure with contrast medium to minimize the risk of acute renal failure and lactic acidosis. Alcohol consumption must be carefully managed and depends on amount, timing, and type of medication. It is never the recommendation to double the medication. Medication should be continued during illness, with close glucose and ketone monitoring. The α-glucosidase inhibitors ("starch blockers" such as acarbose [Precose] and miglitol [Glyset]) work by slowing absorption of carbohydrates in the small intestine. They should be taken with the first bite of each main meal.

Group of hormones that oppose effects of insulin

Counterregulatory hormones

Which statement would be correct for a patient with type 2 diabetes who was admitted to the hospital with pneumonia? A. The patient must receive insulin therapy to prevent ketoacidosis B. The patient has islet cell antibodies that have destroyed the pancreas's ability to produce insulin. C. The patient has minimal or absent endogenous insulin secretion and requires daily insulin injections D. The patient may have sufficient endogenous insulin to prevent ketosis but is at risk for hyperosmolar hyperglycemic syndrome

D. The patient may have sufficient endogenous insulin to prevent ketosis but is at risk for hyperosmolar hyperglycemic syndrome Rationale. Hyperosmolar hyperglycemic syndrome (HHS) is a life-threatening syndrome that can occur in a patient with diabetes who is able to produce enough insulin to prevent diabetic ketoacidosis (DKA) but not enough to prevent severe hyperglycemia, osmotic diuresis, and extracellular fluid depletion.

Major complication of type 1 diabetes

DKA (Diabetic Ketoacidosis) - Profound insulin deficiency characterized by hyperglycemia, ketosis, acidosis and dehydration

Characterized by hyperglycemia upon awakening. Suggested that the growth hormone and cortisol are excreted increased amounts in the early AM. It is treated by increasing the insulin dosage or an adjustment in administration time.

Dawn Phenomenon

Glucose levels usually return to normal at 10 weeks post partum True or False

False. Glucose levels usually return to normal at 6 weeks post partum

Diabetes that develops during the 2nd trimester of pregnancy

Gestational Diabetes

This diagnostic test can be tested accurately every 3 months and it depends solely on the blood glucose level.

HG A1C

Major complication in type 2 diabetes

HHS Hyperosmolar Hyperglycemic Syndrome life-threatening syndrome ; able to produce enough insulin to prevent DKA but not enough to prevent hyperglycemia, osmotic diuresis and extracellular depletion; less common than DKA

Examples of Rapid Acting Insulin Medication

Humalog (lispro) NovoLog (aspart)

Examples of Short Acting Medication

Humulin R (Regular) Novolin R (Regular)

what do Glucagon, Epinephrine, Cortisol, and Growth Hormone do to blood sugar?

Increase it

Decreases blood glucose levels Insulin or Glucagon?

Insulin

Promotes glucose transport into the cell Insulin or Glucagon?

Insulin

Oral agents works on these three defects of type 2 diabetes

Insulin resistance Decreased insulin production Increased hepatic glucose production

Examples of Intermediate acting

NPH ( Humulin N, Novolin N)

What is the onset, peak and duration of Intermediate-acting insulin?

Onset: 2-4 hours Peak: 4-10 hours Duration: 10-16 hours

Classic symptoms of diabetes type 1

Polyuria (excessive urination) Polydipsia (Excessive thirst) Polyphasia (Excessive hunger)

Blood glucose high but not high enough to be diagnosed as having diabetes

Prediabetic

Insulin is a hormone produced by______ cells in the islets of ___________of the __________

Produced by the B cells in the Islets of Langerhans of the pancreas

what is the only type insulin that can be adminstered IV?

Regular

This insulin is the only type that can be mixed with other insulins

Regular (NPH)

This type of diabetes results from another medical condition or medication to treat a condition

Secondary diabetes

A rebound effect. Hyperglycemia in the AM. A high dose of insulin produces a decline in the blood glucose levels in the night furthermore resulting in the release of counter regulatory hormones. These hormones stimulate lipolysis, gluconeogenesis, and glycogenolysis which produces rebound hyperglycemia. Treated by decreasing the bedtime insulin or a bedtime snack.

The Somogyi effect

Rapid Acting (bolus):time before meal, onset, peak, duration

Time - Injected 0-15 minutes before meal Onset- 15 minutes Peak- 60-90 minutes Duration- 3-4 hours

Short Acting (bolus): time before meal, onset, peak, duration

Time: Injected 30-40 minutes before meal Onset: 30 min- 1 hour Peak: 2-3 hours Duration: 3-6 hours

In-use vials may be left at room temperature up to 4 weeks. True or false?

True

Prediabetics may have long term damage already occurring in the body. True or False

True

Rotate injections within one particular site. True or false?

True

Caused by genetic predisposition (HLAs) and exposure to a virus Type 1 or Type 2?

Type 1

Progressive destruction of beta cells by body's own T cells, Type 1 ot Type 2?

Type 1

Hypoglycemia symptoms

Weakness visual disturbances confusion, irritability, diaphoresis, tremors, hunger

Signs and symptoms are usually abrupt for this type of diabetes.

type 1 diabetes

This type of diabetes has Abnormal or No production of insulin and needs injections?

type 1 diabetes

This type of diabetes is a condition where the body can not fully utilize the insulin production and/or there is abnormal/little to no insulin produced in the body

type 2 diabetes

A patient is admitted with diabetes mellitus, malnutrition, and cellulitis. The patient's potassium level is 5.6 mEq/L. Which factors could contribute to this laboratory result (select all that apply)? A. The level may be increased as a result of dehydration that accompanies hyperglycemia. B. The patient may be excreting extra sodium and retaining potassium because of malnutrition. C. The level is consistent with renal insufficiency that can develop with renal nephropathy. D. The level may be raised because of metabolic ketoacidosis caused by hyperglycemia. E. The level may be raised because excess insulin is being given and causing potassium shifts.

A. The level may be increased as a result of dehydration that accompanies hyperglycemia. C. The level is consistent with renal insufficiency that can develop with renal nephropathy. D. The level may be raised because of metabolic ketoacidosis caused by hyperglycemia. Rationale. Malnutrition does not cause sodium excretion accompanied by potassium retention; it is not a contributing factor to this patient's potassium level. The additional stress of cellulitis may lead to an increase in the patient's serum glucose levels. Dehydration may cause hemoconcentration, resulting in elevated serum readings. Kidneys may have difficulty excreting potassium if renal insufficiency exists. You must consider the potential for metabolic ketoacidosis because potassium leaves the cell when hydrogen enters in an attempt to compensate for a low pH.

12. A patient is admitted with diabetes mellitus, malnutrition, and cellulitis. The patient's potassium level is 5.6 mEq/L. The nurse understands that what could be contributing factors for this laboratory result (select all that apply)? A. The level may be increased as a result of dehydration that accompanies hyperglycemia. B. The patient may be excreting extra sodium and retaining potassium because of malnutrition. C. The level is consistent with renal insufficiency that can develop with renal nephropathy. D. The level may be raised as a result of metabolic ketoacidosis caused by hyperglycemia. E. This level demonstrates adequate treatment of the cellulitis and effective serum glucose control.

A. The level may be increased as a result of dehydration that accompanies hyperglycemia. C. The level is consistent with renal insufficiency that can develop with renal nephropathy D. The level may be raised as a result of metabolic ketoacidosis caused by hyperglycemia. Rationale. The additional stress of cellulitis may lead to an increase in the patient's serum glucose levels. Dehydration may cause hemoconcentration, resulting in elevated serum readings. Kidneys may have difficulty excreting potassium if renal insufficiency exists. Finally, the nurse must consider the potential for metabolic ketoacidosis since potassium will leave the cell when hydrogen enters in an attempt to compensate for a low pH. Malnutrition does not cause sodium excretion accompanied by potassium retention. Thus it is not a contributing factor to this patient's potassium level. The elevated potassium level does not demonstrate adequate treatment of cellulitis or effective serum glucose control.

The patient with diabetes and shortness of breath is brought from the nursing home to the hospital emergency department. The electrocardiogram (ECG) shows evidence of a myocardial infarction (MI), but the patient denied ever having chest pain. Which is the best explanation of what happened? A. The patient had a "silent" MI related to autonomic neuropathy. B. The patient had chest pain but forgot because of dementia. C. The patient minimized the chest pain because he was worried about costs. D. The patient has the psychologic defense mechanism of denial.

A. The patient had a "silent" MI related to autonomic neuropathy. Rationale. Cardiovascular abnormalities associated with autonomic neuropathy include painless myocardial infarction. Shortness of breath related to decreased cardiac functioning can be the first overt sign or symptom. Patients with dementia accurately report chest pain at the time it is occurring when asked, although they may later fail to recall it. Defense mechanisms should not be assumed without further evidence.

Which statement is true regarding the difference between type 1 and type 2 diabetes mellitus? A. Type 2 diabetes has decreased insulin secretion and increased insulin resistance. B. Type 2 diabetes has a total dependency on an outside source of insulin. C. Type 1 diabetes typically occurs in older, obese adults. D. Type 1 diabetes can result in hyperosmolar hyperglycemic syndrome (HHS).

A. Type 2 diabetes has decreased insulin secretion and increased insulin resistance. Rationale. In type 2 diabetes mellitus, secretion of insulin by the pancreas is reduced, and the cells become resistant to insulin. Patients with type 2 diabetes retain the ability to make insulin, although in inadequate amounts. The patient with type 1 diabetes is typically a thin, younger person. An uncontrolled type 1 diabetic may progress to a state of diabetic ketoacidosis. Type 2 diabetes can advance to hyperosmolar hyperglycemic syndrome (HHS).

The patient has vision problems. What intervention can help the patient independently manage her insulin administration? A. Use an insulin pen, listening to the clicks. B. Have family members prefill syringes for a month ahead of time. C. Ask the physician to change the prescription to oral insulin. D. Mix the basal insulin with rapid-acting insulin in the same syringe.

A. Use an insulin pen, listening to the clicks. Rationale. Patients with poor vision cannot see markings on a syringe but can count the audible clicks of the pen. Prefilled syringes are stable only for a week when stored in the refrigerator. Although oral insulin is a term used by lay people, it is inaccurate. Insulin is inactivated by gastric juices; the oral antidiabetic medications stimulate the body's production of and sensitivity to insulin and are not interchangeable with insulin. Basal insulin cannot be mixed with any other insulin or solution

Which are appropriate therapies for patients with diabetes mellitus? (select all that apply) A. Use of statins to treat dyslipidemia B. Use of diuretics to treat nephropathy C. Use of ACE inhibitors to treat nephropathy D. Use of serotonin agonists to decrease appetite E. Use of laser photocoagulation to treat retinopathy

A. Use of statins to treat dyslipidemia C. Use of ACE inhibitors to treat nephropathy E. Use of laser photocoagulation to treat retinopathy Rationale. In patients with diabetes who have microalbuminuria or macroalbuminuria, angiotensin-converting enzyme (ACE) inhibitors (e.g., lisinopril [Prinivil, Zestril]) or angiotensin II receptor antagonists (ARBs) (e.g., losartan [Cozaar]) should be used. Both classes of drugs are used to treat hypertension and have been found to delay the progression of nephropathy in patients with diabetes. The statin drugs are the most widely used lipid-lowering agents. Laser photocoagulation therapy is indicated to reduce the risk of vision loss in patients with proliferative retinopathy, in those with macular edema, and in some cases of nonproliferative retinopathy.

salicylic acid, found in many corn plasters

CHemical irritation that can cause foot ulcers are caused by

until the sodium bicarb level reaches 15-18 mEq/L and glucose is normal

As long as what happens do you have to continue the insulin drip?

A diabetic client has a glycosylated hemoglobin (HbA1C) level of 9.4%. What does the nurse say to the client regarding this finding? "Keep up the good work." "This is not good at all." "What are you doing differently?" Correct "You need more insulin."

Assessing the client's regimen or changes he or she may have made is the basis for formulating interventions to gain control of blood glucose. HbA1C levels for diabetic clients should be less than 7%; a value of 9.4% shows poor control over the past 3 months. Telling the client this is not good, although true, does not take into account problems that the client may be having with the regimen and sounds like scolding. Although it may be true that the client needs more insulin, an assessment of the client's regimen is needed before decisions are made about medications.

Which nursing action can the home health nurse delegate to a home health aide who is making daily visits to a client with newly diagnosed type 2 diabetes? Assist the client's spouse in choosing appropriate dietary items. Evaluate the client's use of a home blood glucose monitor. Inspect the extremities for evidence of poor circulation. Assist the client with washing the feet and applying moisturizing lotion. Correct

Assisting with personal hygiene is included in the role of home health aides. Assisting with dietary choices, evaluating the effectiveness of teaching, and performing assessments are complex actions that should be implemented by licensed nurses.

the peripheral, carotid, cerebral, and coronary vessels.

Atherosclerosis plaque development occurs earlier and usually affects

sympathetic and parasympathetic nervous systems

Autonomic neuropathy affects the

pupillary response, and functions of the heart, gi, and gu systems.

Autonomic neuropathy affects what parts of the body?

Which client does the nurse caution to avoid self-monitoring of blood glucose (SMBG) at alternate sites? a. 75-year-old client whose blood glucose levels show little variation b. 55-year-old client who has hypoglycemic unawareness c. 80-year-old client with type 2 diabetes mellitus d. 45-year-old client with type 1 diabetes mellitus

B Comparison studies have shown wide variation between fingertip and alternate sites, and variation is most evident during times when blood glucose levels are rapidly changing. Teach patients that there is a lag time for blood glucose levels between the fingertip and other sites when blood glucose levels are changing rapidly and that the fingertip reading is the only safe choice at those times. Because of this lag time, clients who have hypoglycemic unawareness should never use alternate sites for SMBG. Cognitive Level: Applying or Higher Client Needs Category: Health Promotion and Maintenance Nursing Process Step: Implementation

A client with type 1 diabetes mellitus received regular insulin at 7:00 a.m. The client will need to be monitored for hypoglycemia at which time? A. 7:30 a.m. B. 11:00 a.m. C. 2:00 p.m. D. 7:30 p.m.

B Regular insulin is a short-acting type of insulin. Onset of action to regular insulin is ½ to 1 hour. The peak effect time is when hypoglycemia may start to occur. Peak time for regular insulin is 2-4 hours. Therefore, 11:00 a.m. is the anticipated peak time for regular insulin received at 7:00 a.m. The other options for peak times for regular insulin are incorrect.

The nurse is beginning to teach a diabetic patient about vascular complications of diabetes. What information is appropriate for the nurse to include? A. Macroangiopathy does not occur in type 1 diabetes but rather in type 2 diabetics who have severe disease. B. Microangiopathy is specific to diabetes and most commonly affects the capillary membranes of the eyes, kidneys, and skin. C. Renal damage resulting from changes in large- and medium-sized blood vessels can be prevented by careful glucose control. D. Macroangiopathy causes slowed gastric emptying and the sexual impotency experienced by a majority of patients with diabetes

B. Microangiopathy is specific to diabetes and most commonly affects the capillary membranes of the eyes, kidneys, and skin. Rationale. Microangiopathy occurs in diabetes mellitus. When it affects the eyes, it is called diabetic retinopathy. When the kidneys are affected, the patient has nephropathy. When the skin is affected, it can lead to diabetic foot ulcers. Macroangiopathy can occur in either type 1 or type 2 diabetes and contributes to cerebrovascular, cardiovascular, and peripheral vascular disease. Sexual impotency and slowed gastric emptying result from microangiopathy and neuropathy.

18. A patient with diabetes mellitus who has multiple infections every year needs a mitral valve replacement. What is the most important preoperative teaching the nurse should provide to prevent a cardiac infection postoperatively? A. Avoid sick people and wash hands. B. Obtain comprehensive dental care. C. Maintain hemoglobin A1 c below 7%. D. Coughing and deep breathing with splinting

B. Obtain comprehensive dental care. Rationale. A person with diabetes is at high risk for postoperative infections. The most important preoperative teaching to prevent a postoperative infection in the heart is to have the patient obtain comprehensive dental care because the risk of septicemia and infective endocarditis increases with poor dental health. Avoiding sick people, hand washing, maintaining hemoglobin A1c below 7%, and coughing and deep breathing with splinting would be important for any type of surgery, but not the priority with mitral valve replacement for this patient.

You should suspect an eating disorder in a woman with type 1 diabetes when learning what information? A. Eats 1 cup of ice cream every night before going to bed B. Skips insulin doses to cause rapid weight loss C. Gained a pound per month despite indicating she follows her diet D. Admits to routinely skipping breakfast

B. Skips insulin doses to cause rapid weight loss Rationale. Women with diabetes and an eating disorder intentionally decrease their dose of insulin or omit the dose to result in a rapid weight loss through calorie purging. The hyperglycemia leads to glycosuria, which promotes weight loss of calories as glucose in the urine. The other options are less than optional behaviors but do not signal an eating disorder.

What is a typical finding of hyperosmolar hyperglycemic syndrome (HHS)? A. Occurs in type 1 diabetes as the presenting symptom B. Slow onset resulting in a blood glucose level greater than 600 mg/dL C. Ketone bodies higher than 4+ in urine D. Signs and symptoms of diabetes insipidus

B. Slow onset resulting in a blood glucose level greater than 600 mg/dL Rationale. HHS has a slower onset than diabetic ketoacidosis. HHS is often related to impaired thirst sensation, inadequate fluid intake, or functional inability to replace fluids. Because of the slower onset, the blood glucose levels can be quite high (more than 600 mg/dL) before diagnosis. HHS is seen in type 2 diabetics, and there is enough circulating insulin to prevent ketoacidosis. Diabetes insipidus is related to inadequate antidiuretic hormone secretion or kidney response with dilute, frequent urination. It is not related to HHS.

Which elevated laboratory finding is the best indication of potential diabetic nephropathy? A. Blood urea nitrogen (BUN) level B. Urine albumin-to-creatinine ratio C. Urine specific gravity D. Chloride (Cl-) level

B. Urine albumin-to-creatinine ratio Rationale. Screening for nephropathy depends on the urinary albumin-to-creatinine ratio and a serum creatinine level. (Creatinine indicates how well the kidney is filtering). BUN alone, without correction to creatinine, can indicate many other issues, including dehydration and liver function. Unless there is renal failure, urine specific gravity is more indicative of dehydration. Chloride is not a direct indicator of kidney function.

Polydipsia and Polyuria related to diabetes mellitus are primarily due to: A. the release of ketones from cells during fat metabolism B. fluid shifts resulting from the osmotic effect of hyperglycemia C. damage to the kidneys from exposure to high levels of glucose D. changes in RBCs resulting from attachment of excessive glucose to hemoglobin

B. fluid shifts resulting from the osmotic effect of hyperglycemia Rationale. The osmotic effect of glucose produces the manifestations of polydipsia and polyuria.

The polydipsia and polyuria related to diabetes mellitus are primarily caused by the A. release of ketones from cells during fat metabolism. B. fluid shifts resulting from the osmotic effect of hyperglycemia. C. damage to the kidneys from exposure to high levels of glucose. D. changes in red blood cells resulting from attachment of excessive glucose to hemoglobin.

B. fluid shifts resulting from the osmotic effect of hyperglycemia. Rationale. The osmotic effect of glucose produces the manifestations of polydipsia and polyuria.

1. A 54-year-old patient admitted with type 2 diabetes asks the nurse what "type 2" means. What is the most appropriate response by the nurse? A. "With type 2 diabetes, the body of the pancreas becomes inflamed." B. "With type 2 diabetes, insulin secretion is decreased, and insulin resistance is increased." C."With type 2 diabetes, the patient is totally dependent on an outside source of insulin." D. "With type 2 diabetes, the body produces autoantibodies that destroy β-cells in the pancreas."

B."With type 2 diabetes, insulin secretion is decreased, and insulin resistance is increased." Rationale: In type 2 diabetes mellitus, the secretion of insulin by the pancreas is reduced, and/or the cells of the body become resistant to insulin. The pancreas becomes inflamed with pancreatitis. The patient is totally dependent on exogenous insulin and may have had autoantibodies destroy the β-cells in the pancreas with type 1 diabetes mellitus.

Which precaution is most important for the nurse to teach a client who has cardiovascular autonomic neuropathy (CAN) from diabetes? a. "Avoid drinking ice-cold beverages." b. "Be sure to check your blood pressure twice daily." c. "Change positions slowly when moving from sitting to standing." d. "Check your hands and feet weekly for areas of numbness or sensation change."

C Cardiovascular autonomic neuropathy (CAN) affects sympathetic and parasympathetic nerves of the heart and blood vessels. This problem contributes to left ventricular dysfunction, painless myocardial infarction, and exercise intolerance. Most often, CAN leads to orthostatic (postural) hypotension and syncope (brief loss of consciousness on standing). These problems are from failure of the heart and arteries to respond to position changes by increasing heart rate and vascular tone. As a result, blood flow to the brain is interrupted briefly. Orthostatic hypotension and syncope increase the risk for falls, especially among older adults. Although checking blood pressure twice daily is helpful, it does not prevent orthostatic hypotension, nor is there any guarantee that such hypotension will occur during blood pressure measurement. Sensation changes are associated with peripheral neuropathy, not cardiovascular autonomic neuropathy. Avoiding cold beverages is no longer a recommended action. Cognitive Level: Applying or Higher Client Needs Category: Health Promotion and Maintenance Nursing Process Step: Implementation

The nurse in the endocrine clinic is reviewing type 1 and type 2 diabetes with a group of nursing students. Which explanation by the students indicates their understanding of the types of diabetes? A. Most clients with type 1 diabetes are born with it. B. People with type 1 diabetes are often obese. C. Those with type 2 diabetes make insulin, but in inadequate amounts. D. People with type 2 diabetes do not develop typical diabetic complications

C he explanation by the students that indicate understanding of the type of diabetes is "Those with type 2 diabetes make insulin, but in inadequate amounts." People with type 2 diabetes may also have resistance to existing insulin. Most clients with type 1 diabetes are not born with it. Although type 1 diabetes may occur early in life, it is considered an autoimmune disorder in which beta cells are destroyed in a genetically susceptible person. Risk for type 1 DM is determined by inheritance of genes coding for the HLA-DR and HLA-DQA and DQB tissue types (McCance et al., 2014). However, inheritance of these genes only increases the risk, and most people with these genes do not develop type 1 DM. Obesity is typically associated with type 2 diabetes. People with type 2 diabetes are at risk for typical diabetic complications, especially cardiovascular diseases.

Which symptoms reported by a patient with diabetes mellitus are most important to follow-up? A. "My vision has been getting fuzzier over the past year." B. "I cannot read the small print anymore." C. "There is something like a veil of blackness coming across my vision." D. "I have yellow discharge from one eye."

C. "There is something like a veil of blackness coming across my vision." Rationale. Diabetic retinopathy, particularly proliferative retinopathy, can cause retinal detachment, which has the classic new symptom of a veil coming across the field of vision. This requires emergency treatment. Chronic blurry vision can be cataracts and is not emergent. Change in the ability to read things near to the eye (presbyopia or farsightedness) is an age-related change and not emergent. Conjunctivitis needs treatment but is not as emergent as retinal detachment.

The patient has type 1 diabetes mellitus and is found unresponsive with cool and clammy skin. What action is a priority? A. Obtain a serum glucose level. B. Give hard candy under the tongue. C. Administer glucagon per standing order. D. Notify the health care provider.

C. Administer glucagon per standing order. Rationale. The patient has signs and symptoms of hypoglycemia for which treatment should be the priority. Glucagon stimulates a strong hepatic response to convert glycogen to glucose and therefore makes glucose rapidly available. Waiting for a serum result (up to an hour) is improper because brain cells continue to die from a lack of glucose. Nothing solid should be placed in the mouth when the patient has an altered level of consciousness and can aspirate. With obvious symptoms, emergent treatment takes priority over notifying the health care provider.

15. A 65-year-old patient with type 2 diabetes has a urinary tract infection (UTI). The unlicensed assistive personnel (UAP) reported to the nurse that the patient's blood glucose is 642 mg/dL and the patient is hard to arouse. When the nurse assesses the urine, there are no ketones present. What collaborative care should the nurse expect for this patient? A. Routine insulin therapy and exercise B. Administer a different antibiotic for the UTI. C. Cardiac monitoring to detect potassium changes D. Administer IV fluids rapidly to correct dehydration.

C. Cardiac monitoring to detect potassium changes Rationale. This patient has manifestations of hyperosmolar hyperglycemic syndrome (HHS). Cardiac monitoring will be needed because of the changes in the potassium level related to fluid and insulin therapy and the osmotic diuresis from the elevated serum glucose level. Routine insulin would not be enough, and exercise could be dangerous for this patient. Extra insulin will be needed. The type of antibiotic will not affect HHS. There will be a large amount of IV fluid administered, but it will be given slowly because this patient is older and may have cardiac or renal compromise requiring hemodynamic monitoring to avoid fluid overload during fluid replacement.

What priority action should you take if the patient with type 2 diabetes complains of blurred vision and irritability? A. Call the physician. B. Administer insulin as ordered. C. Check his blood glucose level. D. Assess for other neurologic symptoms.

C. Check his blood glucose level. Rationale. Blood glucose testing should be performed whenever hypoglycemia is suspected so that immediate action can be taken if necessary.

8. A patient, who is admitted with diabetes mellitus, has a glucose level of 380 mg/dL and a moderate level of ketones in the urine. As the nurse assesses for signs of ketoacidosis, which respiratory pattern would the nurse expect to find? A. Central apnea B. Hypoventilation C. Kussmaul respirations D. Cheyne-Stokes respirations

C. Kussmaul respirations Rationale. In diabetic ketoacidosis, the lungs try to compensate for the acidosis by blowing off volatile acids and carbon dioxide. This leads to a pattern of Kussmaul respirations, which are deep and nonlabored. Central apnea occurs because the brain temporarily stops sending signals to the muscles that control breathing, which is unrelated to ketoacidosis. Hypoventilation and Cheyne-Stokes respirations do not occur with ketoacidosis.

The patient has a history of poor control of her diabetes mellitus. When you ask the patient how the diet, insulin administration, and glucose testing are going, she responds with a flat affect and says, "I don't know." Which factor is most important for you to assess? A. Dementia B. Lack of resources C. Depression D. Lack of knowledge

C. Depression Rationale. Consistent "I don't know" signals "I don't care" and depression. Patients with diabetes have high rates of psychiatric disorders, particularly depression, which contributes to poor adherence, feelings of helplessness, and poor outcomes. Patients need to be assessed for the signs and symptoms of depression at each visit with a health care professional. Patients with dementia forget but care enough to answer. Depression should be ruled out before dealing with a lack of resources or lack of knowledge to use them.

The patient had a hypoglycemic episode and is treated with a concentrated glucose oral tablet. Fifteen minutes later the capillary glucose level (Accu-Check) is 150 mg/dL. What action should you take? A. Administer a second bolus of glucose solution. B. Administer regular insulin per sliding scale. C. Have the patient eat peanut butter and toast. D. Obtain a serum glucose level.

C. Have the patient eat peanut butter and toast Rationale. The patient has had an appropriate response to the glucose. Now a complex carbohydrate is needed to prevent hypoglycemia from reoccurring. There is no need for a second bolus of glucose because the result is within normal range. Insulin is not given, even though the glucose level is slightly elevated. The short-acting glucose is metabolized and insulin administration can increase the risk of a second hypoglycemic reaction. A serum confirmation of the level can be obtained but is not the priority.

Which lower extremity or foot finding is a sign of sensory neuropathy in a patient with diabetes mellitus? A. Dusky when legs are dependent B. Pitting pedal edema C. Intermittent claudication D. Strong pedal pulse

C. Intermittent claudication Rationale. Peripheral arterial disease (PAD) is caused by a reduction of blood flow to the lower extremities. Classic signs include intermittent claudication, pain at rest, cold feet, loss of hair, delayed capillary filling, and dependent rubor. Dusky legs when they are dependent, pitting pedal edema, and a strong pedal pulse are signs of peripheral venous disease.

The male patient with diabetes and heart disease confides to you that he can no longer have an erection. What is the reason for these changes? A. It is a normal part of aging and is relieved with sildenafil (Viagra). B. It usually is related to emotions and is a temporary problem. C. It is often the first sign of diabetic autonomic neuropathy. D. It indicates that the patient has developed a neurogenic bladder.

C. It is often the first sign of diabetic autonomic neuropathy. Rationale. Erectile dysfunction (ED) is common and often is the first manifestation of autonomic failure. ED is not a normal part of aging, and sildenafil cannot be effective if the problem is related to nerve transmission. Sildenafil should not be taken if the patient is on nitrates. ED is a common long-term complication of diabetes. Neurogenic bladder is related to urinary retention.

The patient with diabetes reports tingling and burning in the lower extremities at night. The patient asks you why the primary health care provider prescribed the selective serotonin reuptake inhibitor (SSRI) duloxetine (Cymbalta). What is the best response? A. The doctor thought the discomfort was causing the patient to be depressed. B. The drug is known to improve patients' moods and enhance coping. C. It regulates pain by affecting neurotransmitters that transmit pain through the spine. D. It deadens the sensitivity to peripheral nerve endings.

C. It regulates pain by affecting neurotransmitters that transmit pain through the spine. Rationale. SSRI drugs work by inhibiting the reuptake of norepinephrine and serotonin, which are neurotransmitters that are believed to play a role in the transmission of pain through the spinal cord. Duloxetine is thought to relieve pain by increasing the levels of serotonin and norepinephrine, which improves the body's ability to regulate pain. The drugs are not given specifically for depression, although it may have a mood-enhancing effect. Duloxetine does not deaden the sensitivity of peripheral nerve endings.

cold but sensation is normal. When the foot is raised it becomes pail, but when it's lowered it becomes red. There is often weak or absent pulses

In neuropathy when the blood supply is impaired but neurological function is adequate the foot is

A diabetic patient has a serum glucose level of 824mg/dL (45.7 mol/dL) and is unresponsive. After assessing the patient, the nurse suspects diabetic ketoacidosis rather than hyperosmolar hyperglycemic syndrome based on the finding of A. polyuria B. Severe hydration C. Rapid, deep respirations D. Decreased serum potassium

C. Rapid, deep respirations Rationale. Signs and symptoms of DKA include manifestations of dehydration, such as poor skin turgor, dry mucous membranes, tachycardia, and orthostatic hypotension. Early symptoms may include lethargy and weakness. As the patient becomes severely dehydrated, the skin becomes dry and loose, and the eyeballs become soft and sunken. Abdominal pain is another symptom of DKA that may be accompanied by anorexia and vomiting. Kussmaul respirations (i.e., rapid, deep breathing associated with dyspnea) are the body's attempt to reverse metabolic acidosis through the exhalation of excess carbon dioxide. Acetone is identified on the breath as a sweet, fruity odor. Laboratory findings include a blood glucose level greater than 250 mg/dL, arterial blood pH less than 7.30, serum bicarbonate level less than 15 mEq/L, and moderate to high ketone levels in the urine or blood.

16. The newly diagnosed patient with type 2 diabetes has been prescribed metformin (Glucophage). What should the nurse tell the patient to best explain how this medication works? A. Increases insulin production from the pancreas. B. Slows the absorption of carbohydrate in the small intestine. C. Reduces glucose production by the liver and enhances insulin sensitivity. D. Increases insulin release from the pancreas, inhibits glucagon secretion, and decreases gastric emptying.

C. Reduces glucose production by the liver and enhances insulin sensitivity. Rationale. Metformin is a biguanide that reduces glucose production by the liver and enhances the tissue's insulin sensitivity. Sulfonylureas and meglitinides increase insulin production from the pancreas. α-glucosidase inhibitors slow the absorption of carbohydrate in the intestine. Glucagon-like peptide receptor agonists increase insulin synthesis and release from the pancreas, inhibit glucagon secretion, and decrease gastric emptying.

Which presentation would make the clinic nurse suspicious that the patient has undiagnosed diabetes mellitus? A. Has a throat infection that is resistant to the prescribed antibiotic B. History of dry socket syndrome after a tooth extraction C. Relates a history of recurring vaginal yeast infections D. History of sensitivity to alcohol ingestion

C. Relates a history of recurring vaginal yeast infections Rationale. A patient with diabetes mellitus is more susceptible to infections than other patients. The mechanisms for this phenomenon include a defect in the mobilization of inflammatory cells and an impairment of phagocytosis by neutrophils and monocytes. Recurring or persistent infections such as Candida albicans, as well as boils and furuncles, in the undiagnosed patient often lead the health care provider to suspect diabetes. The other options do not signal diabetes mellitus.

The patient with diabetes mellitus confides to you that he is afraid he will lose his vision like his friend did. What is the best response? A. Encourage the patient to look at the blessings he has instead of worrying. B. Verify there is nothing that can influence that outcome but new measures to cope are being developed. C. Stress that tight glycemic control can minimize microvascular complications such as retinopathy. D. Remind the patient that every person is different.

C. Stress that tight glycemic control can minimize microvascular complications such as retinopathy. Rationale. Microvascular complications result from thickening of the vessel membrane in the capillaries and arterioles and is in response to chronic hyperglycemia. They are specific to diabetes, and good control can help prevent complications, including retinopathy, which causes blindness. The first option minimizes the patient's concern and gives advice. The second option is inaccurate. The last option is true but minimizes the concern and omits actions the patient can take, such as controlling glucose and seeing an ophthalmologist for early detection of problems.

Purpose for Meglitinides

Increase insulin production from the pancreas

extreme weakness and poor organ function

Changes in protein metabolism lead to

persistent hyperglycemia and aggravated by htn, and smoking

Changes in the capillaries in microvascular are related to

rapid acting, short acting, and long acting

Clear insulins-

glucagon, catecholamines, cortisol, and growth hormone. Read 1064

Counterregulatory hormones that can cause diabetic ketoacidosis

. The laboratory values of a client who has diabetes mellitus include a fasting blood glucose level of 82 mg/dL (mmol/L) and a hemoglobin A1c (A1C) of 5.9%. What is the nurse's interpretation of these findings? a. The client's glucose control for the past 24 hours has been good but the overall control is poor. b. The client's glucose control for the past 24 hours has been poor but the overall control is good. c. The values indicate that the client has poorly managed his or her disease. d. The values indicate that the client has managed his or her disease well.

D Fasting blood glucose levels provide an indication of the client's adherence to drug and nutrition therapy for DM has been for the previous 24 hours. This client's FBG is well within the normal range. A1C provides an indication of general blood glucose control for the past several months because when glucose attaches to hemoglobin, the attachment is permanent for as long as those hemoglobin molecules are present within red blood cells. Normal red blood cell life span is about 120 days. This client's A1C level is within the desirable range, indicating good long-term glucose control as well as short-term control. Cognitive Level: Applying or Higher Client Needs Category: Physiological Integrity Nursing Process Step: Evaluation

The nurse is performing an admission assessment on a 52-year-old client admitted with type 2 diabetes. Lungs clear, Glucose 179 mg/dL (9.9 mmol/L) Right great toe mottled and cold to touch Hemoglobin A1c 6.9% Client states wears eyeglasses to read After completing the above assessment, which complication of diabetes does the nurse report to the primary health care provider? A. Poor glucose control B. Visual changes C. Respiratory distress D. Decreased peripheral perfusion

D A cold, mottled right great toe may indicate arterial occlusion secondary to arterial occlusive disease or embolization. This must be reported to the primary health care provider to avoid potential gangrene and amputation. Although one glucose reading is elevated, the hemoglobin A1c indicates successful glucose control over the past 3 months. After the age of 40, reading glasses may be needed due to difficulty in accommodating to close objects. Lungs are clear and no evidence of distress is noted.

17. The nurse is teaching a patient with type 2 diabetes mellitus about exercise to help control his blood glucose. The nurse knows the patient understands when the patient elicits which exercise plan? A. "I want to go fishing for 30 minutes each day; I will drink fluids and wear sunscreen." B. "I will go running each day when my blood sugar is too high to bring it back to normal." C. "I will plan to keep my job as a teacher because I get a lot of exercise every school day." D. "I will take a brisk 30-minute walk 5 days per week and do resistance training 3 times a week."

D. "I will take a brisk 30-minute walk 5 days per week and do resistance training 3 times a week." Rationale. The best exercise plan for the person with type 2 diabetes is for 30 minutes of moderate activity 5 days per week and resistance training 3 times a week. Brisk walking is moderate activity. Fishing and teaching are light activity, and running is considered vigorous activity.

What is routinely used to diagnose PAD? A. Venous Doppler B. V/Q scan C. D-dimer D. Ankle-brachial index (ABI)

D. Ankle-brachial index (ABI) Rationale. The disease is diagnosed by the medical history, ankle-brachial index (ABI), and angiography. The ABI is calculated by dividing the ankle systolic blood pressure by the highest brachial systolic blood pressure. Venous Doppler is performed for deep vein thrombosis. A V/Q scan is used for pulmonary embolism. D-dimer is used for abnormal clotting effects.

Which assessment is the most sensitive indicator that the IV fluid administration may be too rapid when treating a patient with DKA and a history of renal disease? A. Pedal edema B. Tachypnea C. Urine output of 40 mL/hour D. Change in the level of consciousness

D. Change in the level of consciousness Rationale. Too rapid fluid replacement can lead to hyponatremia and cerebral edema. Pedal edema is a later and relatively insignificant sign. In a bedridden patient, edema is more evident in the sacral area. The Kussmaul respirations are expected; crackles auscultated in the lungs are a more sensitive indicator. The desired urine output for adequate hydration is 30 to 60 mL/hr.

Which finding is the best indication that the patient needs instruction regarding consistent control of her diabetes? A. Fasting serum glucose level is 150 mg/dL. B. Postprandial glucose level is 140 mg/dL. C. Urine ketone level is zero. D. Glycosylated hemoglobin (A1C) level is 9%

D. Glycosylated hemoglobin (A1C) level is 9% Rationale. The glycosylated hemoglobin is the amount of glucose bound to hemoglobin, and it remains there for the life of the cell (120 days). This test indicates glycemic control over 2 to 3 months. The other glucose levels may be temporarily high based on recent intake or indicate prediabetes with impaired glucose tolerance that is not yet diabetes. No ketones indicate good control.

2. The nurse caring for a patient hospitalized with diabetes mellitus would look for which laboratory test result to obtain information on the patient's past glucose control? A. Prealbumin level B. Urine ketone level C. Fasting glucose level D. Glycosylated hemoglobin level

D. Glycosylated hemoglobin level Rationale. A glycosylated hemoglobin level detects the amount of glucose that is bound to red blood cells (RBCs). When circulating glucose levels are high, glucose attaches to the RBCs and remains there for the life of the blood cell, which is approximately 120 days. Thus the test can give an indication of glycemic control over approximately 2 to 3 months. The prealbumin level is used to establish nutritional status and is unrelated to past glucose control. The urine ketone level will only show that hyperglycemia or starvation is probably currently occurring. The fasting glucose level only indicates current glucose control.

Which statement correctly describes a patient with type 2 diabetes who is admitted to the hospital with pneumonia? A. Must receive insulin therapy to prevent the development of ketoacidosis B. Has islet cell antibodies that have destroyed the ability of the pancreas to produce insulin C. Has minimal or absent endogenous insulin secretion and requires daily insulin injections D. May have sufficient endogenous insulin to prevent ketosis but is at risk for hyperosmolar hyperglycemic syndrome (HHS).

D. May have sufficient endogenous insulin to prevent ketosis but is at risk for hyperosmolar hyperglycemic syndrome (HHS). Rationale. HHS is a life-threatening syndrome that can occur in the patient with diabetes who is able to produce enough insulin to prevent diabetic ketoacidosis (DKA) but not enough to prevent severe hyperglycemia, osmotic diuresis, and extracellular fluid depletion.

pelvic girdle and thighs. This causes muscles to look wasted and have involuntary twitching

Diabetic amyotrophy causes pain of the

relative deficiency or absolute deficiency of insulin, in addition to elevated counterregulatory hormones

Diabetic ketoacidosis is a lie threateing emergency caused by a

fasting glucose at 100-125, 2 hour plasma test between 140 and 199, A1c between 5.7 and 6.4

Diagnostic criteria for prediabetes-

high protein, low carb with no more than 100 g/day of carbs.

Diet needs for a hypoglycemic pt

Type 1 Diabetes Mellitus

Disorder of insulin production Diagnosed in childhood or early adulthood insulin dependent. Immune system destroys pancreatic beta cells.

adrenergic

Do adrenergic or neuroglucopenic symptoms appear first in hypoglycemia?

phosphate & mg may lower and ca can rise to dangerous levels

During IV adm of saline and or K+ you must pay careful attention to what minerals?

Contraindications for Metformin

ESRD, alcohol and CT contrast

dehydration, electrolyte imb, and acidosis

Early identification of dka is aimed at correcting 3 main problems

anorexia, HA, and fatigue.

Early signs of diabetic ketoacidosis

warm and pink with good pulses but lacks normal sensation

In neuropathy when the blood flow is adequate the foot is

The nurse is teaching a client with type 2 diabetes about the importance of weight control. Which comment by the client indicates a need for further teaching? "I should begin exercising for at least an hour a day." Correct "I should monitor my diet." "If I lose weight, I may not need to use the insulin anymore." "Weight loss can be a sign of diabetic ketoacidosis."

For long-term maintenance of major weight loss, large amounts of exercise (7 hr/wk) or moderate or vigorous aerobic physical activity may be helpful, but the client must start slowly. Monitoring the diet is key to type 2 diabetes management. Weight loss can minimize the need for insulin and can also be a sign of diabetic ketoacidosis.

constipation, less commonly diarrhea, as well as anorexia, nausea, and vomiting, gastric reflux, and bloating after meals

GI symptoms related to autonomic neuropathy

it must always be diluted

How should potassium be given IV

endogenous, exogenous, and functional.

Hypoglycemia can be classified as

The nurse is providing discharge teaching to a client with newly diagnosed diabetes. Which statement by the client indicates a correct understanding about the need to wear a MedicAlert bracelet? "If I become hyperglycemic, it is a medical emergency." "If I become hypoglycemic, I could become unconscious." Correct "Medical personnel may need confirmation of my insurance." "I may need to be admitted to the hospital suddenly."

Hypoglycemia is the most common cause of medical emergency in clients with diabetes. A MedicAlert bracelet is helpful if the client becomes hypoglycemic and is unable to provide self-care. Hyperglycemia is not a medical emergency unless it is acidosis; people with diabetes tolerate mild hyperglycemia routinely. Insurance information and information needed for hospital admission do not appear on a MedicAlert bracelet.

not have symptoms of hypoglycemia. Mental changes are often the first symptom in these pt's

Hypoglycemic unawareness causes the pt to

type 1

IS ketoacidosis primarily a problem with type 1 or 2 diabetes?

dehydrated, weak, and lethargic with abd pain, nausea, vomiting, fruity breath, inc respirations, tachy, blurred vision, and hypothermia

If DA is left untreated the pt becomes

1 week, and you myst store it in a vertical or tilted position, and pull the syringe back slightly before adm to rock the insulin

If a prefilled syringe is refrigerated how long is it stable?

transient (temp) hyperkalemia

In dka potassium shifts out of cells causing

Amylin Analogs

Meds: Pramlinitide (Syiln Pen) Use: only NON insulin med for type 1 DM

Biguanide

Metformin (Glucophage)

the capillaries to thicken impairing the exchange of nutrients, gases, and wastes

Microvascular affects the small vessels causing

50-70

Moderate hypoglycemia is when blood sugars are

sharp, and stabbing. It's sometimes relieved by walking

Mononeuropathy affects one nerve or one single group of nerves and is describes as

Intermediate Acting Insulin

NPH (Novolin N, Humalin N) Onset: 1.5- 4 Peak: 4-12 hours Duration: 12-18 hours Can mix If giving rapid and NPH, give insulin within 15 minutes before a meal

What insulins are cloudy?

NPH, lispro protamine, and aspart.

In reviewing the health care provider admission requests for a client admitted in a hyperglycemic-hyperosmolar state, which request is inconsistent with this diagnosis? 20 mEq KCl for each liter of IV fluid IV regular insulin at 2 units/hr IV normal saline at 100 mL/hr 1 ampule NaHCO3 IV now Correct

NaHCO3 is given for the acid-base imbalance of diabetic ketoacidosis, not the hyperglycemic-hyperosmolar state, which presents with hyperglycemia and absence of ketosis/acidosis. KCl 20 mEq for each liter of IV fluid will correct hypokalemia from diuresis. IV regular insulin at 2 units/hr will correct hyperglycemia. IV normal saline at 100 mL/hr will correct dehydration.

Genetic Link: Type 2 Diabetes

Not yet understand. Individuals with a first-degree relative with the disease are 10x more likely to develop it.

q 2-4 hours.

On sick days how often should pt's assess their sugar?

hypokalemia that can cause serious arrhythmias.

Once DKA is resolved and K+ is eliminated out of the urine the pt is at risk for

stored fat to provide heat and energy causing polyphagia (hunger)

because most cells aren't able to use glucose with out insulin, the body breaks down

A diabetic patient has the following presentation: unresponsive to voice or touch, tachycardia, diaphoresis, and pallor. Which of the following actions by the healthcare provider is the priority? Choose 1 answer: A. Send blood to the laboratory for analysis B. Administer oxygen per nasal cannula C. Administer 50% dextrose IV per protocol D. Administer the prescribed insulin

c

The blood glucose of a patient who is newly diagnosed with type 1 diabetes mellitus has a blood glucose level of 340 mg/dL. Which type of insulin prescribed for the patient is appropriate to administer at this time? Choose 1 answer: A. NPH + regular (70/30) B. NPH C. Regular D. Glargine

c

Which of these clients with diabetes will the endocrine unit charge nurse assign to an RN who has floated from the labor/delivery unit? a. A client with sensory neuropathy who needs teaching about foot care b. A client with diabetic ketoacidosis who has an IV running at 250 mL/hr c. A client who needs blood glucose monitoring and insulin before each meal d. A client who was admitted with fatigue and shortness of breath

c A nurse from the labor/delivery unit would be familiar with blood glucose monitoring and insulin administration because clients with type 1 and gestational diabetes are frequently cared for in the labor/delivery unit. The clients with sensory neuropathy, diabetic ketoacidosis, and the client with fatigue and shortness of breath all have specific teaching or assessment needs that are better handled by nurses more familiar with caring for adults with diabetes-related complications.

An intensive care client with diabetic ketoacidosis (DKA) is receiving an insulin infusion. When the cardiac monitor shows ventricular ectopy, which assessment will the nurse make? a. Urine output b. 12-lead electrocardiogram (ECG) c. Potassium level d. Rate of IV fluids

c After DKA therapy starts, serum potassium levels drop quickly. An ECG shows conduction changes and ectopy related to alterations in potassium. Hypokalemia is a common cause of death in the treatment of DKA. Detecting and treating the underlying cause of the ectopy is essential. Ectopy is not associated with changes in urine output even though hyperglycemia will cause osmotic diuresis. A 12-lead ECG can verify the ectopy, but the priority is to detect and fix the underlying cause, which is most likely hypokalemia. Increased fluids treat the symptoms of dehydration secondary to DKA, but do not treat the hypokalemia.

A client with typically well controlled diabetes has a glycosylated hemoglobin (HbA1C) level of 9.4%. Which response by the nurse is most appropriate? a. "Keep up the good work." b. "This is not good at all." c. "Have you been doing something differently? d. "You need an increase in your insulin dose."

c The most appropriate response by the nurse is telling the client that the level is high and then assessing the client's regimen or changes he or she may have made. This is the best format to formulate interventions to gain control of blood glucose. HbA1C levels for diabetic clients need to be less than 7%. A value of 9.4% shows poor control over the past 3 months. Telling the client to "keep up the good work" is incorrect. A(HbA1C) level of 9.4% is too high. Scolding the client by saying "this is not good," although true, does not take into account problems the client may be having with the regimen or an undiagnosed illness. Although it may be true that the client needs more insulin, an assessment of the client's regimen is needed before decisions are made about medications.

A client with type 2 diabetes controlled with Metformin is recovering from surgery. The primary health care provider has placed the client on insulin in addition to the metformin. What is the nurse's best response about why the client needs to take insulin? A."Your diabetes is getting worse, so you will need to take insulin." B. "You can't take your metformin while in the hospital." C. Stress, such as surgery, increases blood glucose levels. You'll need insulin to control your blood glucose temporarily." D. "You must take insulin from now on because the surgery will affect your diabetes."

c The nurse's best response is that due to the stress of surgery and NPO status, short-term insulin therapy may be needed perioperatively for clients with diabetes who use oral antidiabetic agents. For those receiving insulin, dosage adjustments may be required until the stress of surgery subsides. No evidence suggests that the client's diabetes has worsened. However, surgery is stressful and may increase insulin requirements. Metformin may be taken in the hospital, but not on days when the client is NPO for surgery. When the client returns to his or her previous health state, oral agents will be resumed.

Humulin N and Novolin N

intermediate acting ex

small aneurysms and hemorrhages in the retina, hard lipid and protein exudates that lea from the blood vessels, infarcted nerve fibers, and changes in retinal veins

manifestations of nonproliferative-

drowsiness, irritability, impairef judgement, blurred vision, slurred speech, ha's, mood swings progressing to disorientation, seizures, and unconsciousness.

neuroglucopenic symptoms

brain

neuroglucopenic symptoms occur when glucose is cut off to the

mono or polyneuropathy, and autonomic neuropathy

neuropathy can be classified as

1-3 hour onset, 6-10 hour peak, and 12-16 hour duration

onset, peak and duration of intermediate acting-

1 hour onset, minimal peak, and 24 hour duration

onset, peak, and duration of long acting-

30min- 1hr onset, 2-4 hour peak, and 5-8 hour duration

onset, peak, and duration of short acting-

Pre-diabetes

potentially reversible stage of the development of type 2 DM FBG 100-125. 2-hour post meal 140-200. A1C 5.7-6.4% condition where blood glucose is higher than normal but below the diagnosis of diabetes

Somogyi Effect

rebound hyperglycemia in mornning a unique combination of hypoglycemia followed by rebound hyperglycemia

Meglitinides

repaglinide nateglinide moa: stimulate insulin secretion "short acting secretagogues" Causes less hypoglycemia than Sulfonylureas

macrovascular

the accelerated atherosclerotic changes associated with diabetes are called

300%

the risk of DM among latinos is _____ that of Caucasian americans

Metabolic syndrome aka syndrome x, insulin resistance syndrome, and cardio vascular dysmetabolic syndrome

what is the common name used to describe a syndrome that is thought to be a precursor of diabetes?


Related study sets

Structures, Locations, and Functions of the Three Types of Muscular Tissue

View Set

1.1 Exercises : Valid or Invalid

View Set

(310) Server Admin Using Microsoft-BPA Nationals Set

View Set

Nutrition chapter 6, 7, 14, 15, 8, 9

View Set

Pennys Book Chapter 2 Review Questions- The Liver

View Set